Вы находитесь на странице: 1из 69

Cat Remembered Questions Part 2:

Day 1: 400 questions split in half with an hour break in the middle
Most of the endo questions were straightforward diagnoses that we learned in class
There were a lot of perio questions
YES there are pics and radiographs on day 1
Read the 72 page document going around bc I realized when trying to remember questions there were easy repeats
from there

Implants- something about what could you do to make sure a buccally angled maxillary implant did not show metal
margin
Lactobacillus plays an important role in caries but not initially
Once enamel cavitation has occurred, the underlying dentin has already been affected by the progression of
the destruction and the lactobacillus organism then becomes a primary agent for further destruction of the
dentin
Strep mutans has been shown to be he most predominant bacteria involved in the initiation of enamel lesion.
Lactobacilli has been shown to be involved in the caries process, but is a secondary organism and flourishes in
a carious environment. Both of these organisms are probably the consequence of a high sugar diet and are
the reason for the dip in pH levels in dental plaque
Flexibility of clasp does not depend on depth of undercut
Polyether impression is the most difficult to take out of the mouth
Material
Advantages
Disadvantages
Reversible
-No custom tray or
-Equipment needed
Hydrocolloids (Agar)
adhesives required
-Dimensionally unstable
-High wettability
-Long working time
-Clean/pleasant with
acceptable odor
Irreversible
-Best wettability*
-Unstable
Hydrocolloids
-Lowest cost*
-Pour immediately
(Alginate)
-Rapid set
-Imbibition
-Worst stability*
-Lowest tear strength*
Polyvinyl Siloxane
-Best dimensional stability*
-Poor wettability
-Pleasant to use
-Release hydrogen gas
-Short setting time
Polysulfide

-Long working time (5-7


minutes)
-Highest tear strength*
-High flexibility
-Good reproduction

Polyether

-Excellent dimensional
stability
-Accurate
-Short setting time
-Good wettability

Condensation Silicone

-Pleasant to use
-Short setting time

Uses
-Multiple impressions
-Problems with moisture

Precautio
-Pour imme
-Pour with

-Most impressions

-Keep imp
damp and
24 hours

-Most impressions

-Delay pou
release of h
-Can delay
1 week
-Pour with
-Allow 10 m

-Messy
-Bad odor
-Stains clothes
-Custom tray required
-Long setting time (8-12
minutes)
-Most Stiff*
-Most expensive*
-Difficult to remove
from mouth
-Unpleasant taste

-Most impressions

Most impressions

-Be careful
teeth when
casts

-Hydrophobic
-Tray requires special
adhesive
-Worst wettability*

-Most impressions

-wait 20 t
before po
stress rel
occur

What material to use on root caries: gerristore and glass ionomer


Characteristics of Gutta percha
Advantages of gutta percha:
o Plasticity: adapts with compaction to irregularities
o Easy to manage
o Little toxicity

o Easy to remove
o Self sterilizing (doesnt support bacterial growth)
Gutta percha has 60-70% zinc oxide
Gutta percha is freely soluble in ether, chloroform, and xylol, and these solvents can be used to remove the
gutta percha during retreatment
There were a handful of pharm definitions- didnt know them. It wasnt the usual efficacy or bioavailability
General principles of drug action
Agonist
Drugs that elicit a response from a tissue are known as agonists. Agonists that produce ceiling effects
(effects that are not exceeded by other drugs are called full agonists). Drugs whose maximal effects
are less than those of full agonists are partial agonists.
Intrinsic
The maximal effect of a drug (emax)
activity
Competiti
A competitive antagonist is a receptor antagonist that binds to a receptor but doesnt activate the
ve
receptor. The antagonist will compete with available agonist for receptor binding sites on the same
antagonis
receptor. Sufficient antagonist will displace the agonist from the binding sites, resulting in a lower
t
frequency of receptor activation. Drugs that bind to a receptor at the same site as the agonist but have
an intrinsic activity of zero (no receptor activation; a=0 are competitive antagonists. By making
receptors less available for agonist binding, a competitive antagonist will depress the response to a
given dose or concentration of agonist.
Affinity
Potency
Efficacy
Therapeut
ic index
Potentiati
on
Synergis
m

Idiosyncra
sy
Occupatio
nal theory
of drug
receptor
interactio
n

Refers to the attractiveness of a drug to its receptor. Affinity is usually measured by the dissociation
constant (Kd). The lower the (kd) the higher the affinity
How much drug does it take to produce an effect
Efficacy is how much of an effect is a drug capable of producing
A comparison of the amount of a therapeutic agent that causes the therapeutic effect to the amount
that causes death or toxicity.. the therapeutic index is the lethal dose of a drug for 50% of the
population (LD50) divided by the minimum effective dose for 50% of the population (ED 50)
Potentiation occurs when two drugs are taken together, and one of them intensifies the action of the
other. This could be expressed by a+b =B. as an example, phenergan , an antihistamine, when
given with a painkilling narcotic such as Demerol intensifies its effect, thereby cutting down on the
amount of narcotic needed
Similar to potentiation. If two drugs are taken together that are similar in action, such as barbiturates
and alcohol, which are both depressants, an effect exaggerated out of proportion to that of each drug
taken separately at the given dose may occur. This could be expressed by 1+1 =5. An example might
be a person taking a dose of alcohol and a dose of a barbiturate. Normally, taken alone, neither
substance would cause serious harm, but if taken together, the combination could cause coma or death
Idiosyncratic reactions are genetically determined abnormal responses to a drug. They are the most
unpredictable in occurrence because the genetically based difference responsible for such a reaction to
a drug may not become evident until the drug is taken for the first time by the patient.
The magnitude of the drug response is proportional to the number of receptors occupied. An
antagonist drug has affinity but no intrinsic activity. The degree of drug action is dependent on the law
of mass action.

In pharm the meds bind to albumin


A lesion that is re-mineralized is more difficult to penetrate in the future
Pan asking what the object was- it was the zygomatic arch

Pan asking what line this was external oblique ridge

1.
2.
3.
4.
5.
6.
7.
8.
9.
10.

maxillary sinus
pterygomaxillary fissure
pterygoid plates
hamulus
zygomatic arch
articular eminence
zygomaticotemporal suture
zygomatic process
external auditory meatus
mastoid process

25. sigmoid notch


26. medial sigmoid depression
27. styloid process
28. cervical vertebrae
29. external oblique ridge
30. mandibular canal
31. mandibular foramen
32. lingula
33. mental foramen
34. submandibular gland fossa

11.
middle cranial fossa
12.
lateral border of the orbit
13.
infraorbital ridge
14.
infraorbital foramen
15.
infraorbital canal
16.
nasal fossa
17.
nasal septum
18.
anterior nasal spine
19.
inferior concha
20.
incisive foramen
21.
hard palate
22.
maxillary tuberosity
23.
condyle
24. coronoid process
Heart rate of 4 year old- I said 110
Age 3
Pulse
110
Systolic BP
100
Diastolic BP
60
Respiratory rate
25

35. internal oblique ridge


36. mental fossa
37. mental ridges
38. genial tubercles
39. hyoid bone
40. tongue
41. soft palate
42. uvula
43. posterior pharyngeal wall
44. ear lobe
45. glossopharyngeal air space
46. nasopharyngeal air space
47. palatoglossal air space

Age 5
100
100
65
20+

Age 12
75
110
70
20-

Adult
70
120
75
15

I had a few question about INR and if you would treat the patient on anticoag. Remember you would treat them and
not take them off meds
What is calculated to develop the normalize the reporting of prothrombin time (PT)? INR
INR is calculated as (PTpatient/PTnormal). True
What are the recommended INR ranges for standard oral anticoagulant therapy? 2.0-3.0
What are the recommended INR ranges for high-dose therapy? 2.5-3.5
I was constantly asked Which of these medications will increase INR?
PT- extrinsic (this is what we are testing in INR)
Normal values for Coagulation
Template Bleeding Time
1 to 9 minutes
Prothrombin Time (PT)
11 to 16 seconds
Partial Thromboplastin Time (PTT)
32-46 seconds
the normal minimum recommended platelet count before surgery is 75,000/mm 3
a normal platelet count is 150,000-450,000/mm3
this test is often used to determine whether oral surgery can be performed safely on a patient taking any oral
anticoagulant. For a patient to be a good candidate for oral surgery, PT time should be within 5-7 seconds of
the control sample
SLOB rule question

Take a Radiograph of a tooth with some sort of lesion overlying it on the film
Take another radiograph of the same tooth after repositioning the x-ray
Object towards the lingual side will appear to shift on the film as the same direction of the repositioned x ray
cone.
I.e. If the x ray cone is shifted mesially , the lingual/palatal root/lesion will shift to the mesial side.
SLOB RULE may determine
o Working length of superimposed canals
o Curvatures of root canals
o Facial-Lingual orientation of instruments or anatomical objects

I had some endo questions about when to use the EPT and how to use (contralateral tooth and adjacent tooth)
Electrical pulp testing simply indicates that there are vital sensory fibers present within the pulp
Electrical pulp testing technique:
o The teeth must be isolated and dried
o The electrode of the pulp tester should be coated with a viscous conductor (ex toothpaste
o The electrode should then be applied to the dry enamel on the middle third of the facial surface of the
crown
o The current flow should be adjusted to increase slowly
o The electrode shouldnt be applied to any restorations (will lead to false reading)
Pulp capping questions (straight forward)
Direct Pulp Capping Indication
1. Small mechanical/traumatic exposure
2. Asymptomatic vital pulp
3. No coronal/periapical pathology

Indirect Pulp Capping Indications


1. Deep carious lesions that encroach the pulp
2. No history of chronic pain
3. No radiographic pathology

4. Vital pulp
5. Normal tooth mobility and color
A pulp-capped tooth should be observed for 3-4 months before a final restoration is placed.
Indications for DPC in immature permanent teeth
o I was specifically asked about DPC for a small carious exposure in an immature tooth. Ive seen
conflicting information about this, but most sources seem to agree that DPC should be reserved for
small iatrogenic or traumatic exposures. Pulpotomy is the best treatment choice for a carious
exposure.
o Note: DPC is NOT indicated for primary teeth (CaOH/MTA can cause internal resorption of primary teeth
rather than reparative dentin formation)
When RCT in max molar what direction would you be most worried about when filing the MB canal?
If a fourth canal is present on the max molar, it will be lingual to the orifice of the MB canal in the MB root
o In the max first molar the fourth canal is midway along the developmental groove connecting the MB
and the palatal canals. This fourth canal is usually small and difficult to instrument and obturate
The most complex root in the entire dentition is the MB of the max molars. They are the largest teeth in the
max arch, the orifice to the canals are the mesiobuccal, the distobuccal, and the palatal. A fourth canal is
often present, and is most often located 1-3 mm lingual to the MB canal
Is NaOCl a chelater? No (this was a these are all properties except.. question)
EDTA, EDTAC, and RC Prep are chelating agents
o EDTA eliminates the smear layer, opens dentinal tubules, and cleans surface for better attachment of
gutta percha and sealer
Chelating agents only act on calcified tissue
NaOCl is an irrigant: irrigants destroy bacteria.
NaOCl acts as an antimicrobial agent, a tissue solvent, and a lubricant
Irrigant
Description
Sodium Hypochlorite
Most widely used irrigant
Hydrogen Peroxide
Use as an alternative. Not as potent as Sodium Hypochlorite
Urea Peroxide (Gly-Oxide)
Best used in narrow or curved canals
What space can premolar get into? Sublingual
Spaces
Maxillary Spaces

Canine Space

Buccal Space

Infratemporal Space
Mandibular Spaces

Buccal space

Submental Space

Submandibular Space

Sublingual Space

Submaxillary Space

Pterygomandibular Space

Masseteric Space

Temporal Space

Masticator Space

Source of Infection

Canines
Maxillary molars and premolars
Maxillary third molars

Mandibular molars and premolars


Mandibular incisors
Mandibular molars and premolars
Mandibular molars and premolars
Mandibular molars
Mandibular molars and premolars
Mandibular third molars
Other spaces
Other spaces

In combined lesion do endo first- the question was more detailed than this though
Know that positive lateral percussion is indicative of perio lesion in combined lesion
COMBINATION THERAPY
In a combined perio-endo lesion, which treatment generally takes precedence? Endodontic Treatment
In what cases should periodontal therapy should be initiated first? Cases with primary periodontal lesions
What is a common clinical finding of a periodontal problem? Pain to lateral percussion on a tooth with a
wide secular pocket
What type of periodontal probing defect may not be managed by endodontic treatment alone? Conical
shaped probing
What type of periodontal lesions can completely heal after endodontic therapy? Blow out and sinus tract
lesions
What types of microorganisms dominate combination lesions? Gram-negative anaerobic bacteria
Know biological width

.97 mm for junctional epithelium (variable)


1.07 for connective tissue attachment
Know where the base of the sulcus is
when measuring the level of attachment, the measurement is take from the junctional epithelium to the CEJ
when measuring pocket depth, the measurement is taken from the JE to the gingival margin
.69
What do you look for at re-eval appointment- color of gingiva, BOP, etc, inflammation etc
I dont remember having any questions on gingival fibers

An increase in GCF is the first detectable sign of inflammation


the source of subgingival calculus is gingival crevicular fluid

Cant remember my flap questions but I feel like there were a lot
Pedicle soft tissue grafts: is a Mucogingival flap designed to serve as a soft tissue graft that maintains an intact
blood supply from the donor site. Pedicle grafts are indicated to widen an inadequate zone of attached gingiva, and to
repair an isolated area of gingival recession. Pedicle grafts offer the best blood supply to the donor tissue
Four types of mucogingival flaps
o Rotated flapaka laterally positioned flaps: uses the donor gingiva from a healthy adjacent tooth to
cover the exposed root of a problem tooth. For deep wide recession
o Advanced flaps (coronally positioned flaps, semilunar flaps) flaps that move vertically in a coronal
direction and dont deviate laterally.

Coronally positioned flap: can be used for class I recession

Semilunar flap: create a pouch for CTG or recession coverage


o Apically positioned flaps

Moves apically and exposes more of the tooth and sometimes the alveolar bone

For increasing the zone of attached gingiva, crown lengthening, and exposing impacted teeth

Replaced flaps: a flap that is reflected and replaced to its original position. Often used for surgical
access or subgingival restoration placement. Can be used for GTR, bone grafting, subgingival surface
Techniques to increase the width of attached gingiva: FGG, CTG, Apically positioned flap
Treating recession: fgg, CTG
Deep/wide recession: laterally positioned flap, subepithelial CTG
Increase zone of attached gingiva: FGG, CTG, apically positioned flap
o

Periodontal Flaps
Full Thickness Mucoperiostreal Flaps

Partial Thickness Flap

Surface Mucosa
o Epithelium
o Basement Membrane
o Connective Tissue & Lamina Propria
Periosteum
Alveolar Bone exposed

Epithelium
Part of Connective Tissue
Periosteum remains in place and alveolar bone is NOT
exposed
Use is indicated when preparing a surgical site for an fgg. Also
used in areas where a dehiscence or fenestration is present

Classification of Periodontal Flaps

Full Thickness (Mucoperiosteal) : All soft tissue including periosteum are reflected exposing alveolar
bone

Partial Thickness (Mucosal): Only epithelium and a layer of connective tissue are reflected. Also called
Split-Thickness Flap

Nondisplaced Flaps: Flap is returned and sutured in its original position

Displaced Flaps: Flaps are placed apically, coronally, or laterally to original position

Conventional Flaps: Interdental papilla is split beneath the contact of the two approximating teeth to
allow reflection of buccal and lingual flaps

Papilla Preservation Flaps: Incorporates the entire papilla in one of the flaps
If have to do surgical extraction on mandibular molar? Hemisect Buccal-lingually
Hemisection: The surgical division of a multirooted tooth (usually a mandibular molar) where a vertical cut is
made through the crown into the furcation and the defective half of the tooth is extracted
o Indicated for class III or IV periodontal furcation defect (also has other indications)
Bicuspidization: a surgical division (also usually involves a mandibular molar) but the crown and root of both
halves are retained.
Root amputation: the removal of one ore more roots of a multirooted tooth. Indicated in cases with a class II or
IV periodontal furcation defect, (many of the same indications as hemisection). Indicated when at least one
root is structurally sound.
Whats the purpose of flap surgery? Access
A couple gingivoplasty and gingivectomy questions (sorry cant remember)
Gingivectomy: an excision of the gingiva. Surgical gingivectomy is for the elimination of suprabony pockets,
gingival enlargements, or suprabony periodontal abscesses. A gingivectomy shouldnt be performed if
osseous recountouring is necessary. A beveled incision is made apical to the pocket depth. The tissue is
removed, the area debrided, and a surgical pack is placed. Healing is by secondary intention with the
formation of a protective clot, epithelial migration, and connective tissue repair. A gingivectomy is only
indicated for patients with redundant or hyperplastic gingival hyperplasia above the alveolar bony crest (ex. In
drug induced gingival hyperplasia, pseudopockets (subrabony pockets))
Gingivoplasty: performed to reshape the tissues where there are deformities like gingival clefts or craters,
gingival enlargements and shelf-like interdental papillae. It is not performed to reduce or eliminate perio
pockets. It can be accomplished with a periodontal knife, scalpel, rotary diamond stone, or electrodes.
If you have suprabony pockets, treat with gingivectomy. If you have infrabony defects, treat with periodontal
flap surgery
Demineralized freeze-dried bone has BMPS
Allograft: DFDBA osteogenic material due to the presence of bone morphogenic proteins that are exposed
during the demineralization process

FDBA is osseoconductive allograft material


Im pretty sure I got a question on what type of bone graft to use for a sinus lift (not sure what I put two of
the answer choices had two grafts each. I picked the only one that had an autogenous graft listed, but I dont
know if its right) .
I had a bunch of questions on diagnosing ANUG, chronic gingivitis, localized etc
NUG vs. NUP
Necrotizing Ulcerative Gingivitis

Interdental Gingival Necrosis: Punched-out papillae

Pain

Bleeding

Fetid oris, Lymphadenopathy, fever, malaise

Associate with treponema denticola (trench mouth)

Necrotizing Ulcerative
Periodontitis

Loss of Clinical attachment


Loss of bone
May result in rapid and extensive necrosis to tissues and underlying
alveolar bone

Darker stain on root seen by recession most likely caused by?


Not sure if this is the answer, but subgingival calculus is often dark because of exposure to gingival crevicular
fluid
Diabetics have 15x the risk of developing perio dz compared to non-diabetic (didnt ask the number just the concept)
Diabetes and perio: diabetes doesnt cause perio disease, but there is evidence that it alters the response of
periodontal tissue to bacterial plaque.
o Poorly controlled diabetic patients often have enlarged gingival, polypoid gingival proliferation, abscess
formation, and loosened teeth. Poorly controlled diabetic patients often have PMNs that demonstrate
impaired chemotaxis, defective phagocytosis, or impaired adherence
o The hyperglycemia that characterizes diabetes leads to nonenzyme glycosylation of proteins and
matrix molecules. These glycolated molecules accumulate in a variety of sites and are called AGEs.
an increase in AGEs may play a role in the progression of perio
a few questions about the order of treatment planning in perio: phase 1 (initial), 2 (surgery), 3 (restorative), 4
(maintenance).
Periodontal Treatment Plan

Preliminary Phase:
o Treat emergencies (pulpal, periodontal, or other)
o Extractions of hopeless teeth

Phase I (Nonsurgical):
o Plaque control and patient education
o Sc/RP
o Caries restorations
o Antimicrobial therapy
o Minor ortho movement/splinting
o Pocket depth and gingival inflammation
o Plaque, calculus, and caries

Phase II (Surgical):
o Periodontal Surgery
o Implants
o Endodontics

Phase III (Restorative):


o Final restorations
o FPD & RPD
o Periodontal examination
o Evaluation of response to restorative procedures

Phase IV(Maintenance):
o Periodic checking:

o
o
o
o

Plaque and calculus


Gingival condition
Occlusion, tooth mobility
Other pathologic changes

There was a question about SCRP and gross debridement how would you go about the first appointment
Nonsurgical phase (phase I therapy)
o Involves removal of calculus and root planning
Had a question about sequencing perio treatments: Plaque control, SRP, restore carious lesions, Perio surgery
o The objective of this phase is to alter or eliminate the microbial etiology and contributing factors to
periodontal diseases, leading to reduction in inflammation. This is achieved by caries control in
patients with rampant caries, removal of calculus, correction of defective restorations, treatment of
carious lesions, and institution of oral hygiene practices. It may include local or systemic antimicrobial
therapy, minor orthodontic tooth movement, occlusal therapy, and provisional splinting and
prostheses. The evaluation phase is designed to determine the effectiveness of the treatment
provided during phase I therapy. It should occur about 4 weeks after the completion of phase I
therapy. His allows time for epithelial and connective tissue healing by the formation of long junctional
epithelium
Know the different names for each brushing technique. They asked me which one was best for interproximal brushing
Electromechanical brushes are more effective than manual brushing. This is particularly true in interproximal
areas
Manual toothbruhsing doesnt generally have much of an effect on interdental plaque and gingivitis
o The most common interdental cleaning aid is dental floss
o Also use floss threader, especially for a patient with a removable prosthesis
o Superflosss: has three portions, a rigid end, a spongy tufted region, and a regular floss part. The rigid
end passes through the embrasure between the retainer and the pontic
Brushing is a supragingival method because the bristles of a brush will only reach a max of 1-2 mm
subgingivally
Bass Method

Angle bristles at 45 from tooth surfaces

This allows bristles to extend into the gingival sulcus when pressure is applied
in a horizontal direction

Start at the most distal tooth in the arch

Use a vibratory, back-and-forth motion to brush

Charters technique

The modified stillman or


roll method

The bristles are perpendicular to the long axis of the teeth. The bristles are
then forced into the interproximal embrasures the bristles of the brush deflect
towards the occlusal surface
In the bass method, the tip enters the sulcus, but in the charters method, it is
good for gingival massage.

The bristles are placed against the teeth, but instead of vibrating the bristles,
the head is rotated so the bristles eventually are pointed towards the occlusal
surfaces in a rolling motion

For oral surgery I had some anatomy questions but they were straightforward like what nerve innervates the soft
palate (prob like 3 or 4 questions of straight anatomy for me)
V2 does sensory to the palate. Tensor veli palatine supplies motor innervation to the palate

10

Cavernous sinus general question: no valves


Which nerves are embedded in the lateral wall of the Cavernous Sinus? CN III,
IV, V1, and V2
What structures pass through the Cavernous Sinus? ICA and CN VI
Cavernous Sinus Syndrome is caused by edema that paralysis? CN III,
CN IV, CN VI, V1 and V2

CN

When removing mandibular tuberosity in danger of hitting lingual nerve


most common cause of paresthesia of the lower lip is removal of a
mandibular third molar
What is an advantage of body osteotomy over BSSO?
Mandible Surgeries
Sagittal Split Osteotomy

Standard procedure in treating:


o Retrognathia

Vertical Ramus Osteotomy

Vertical Body Osteotomy

Step Osteotomy

Prognathia

Position of the condyle is unchanged during the correction of


mandibular prognathism or retrognathism

Used to set the mandible posteriorly

Involves extracting mandibular teeth bilaterally


o Bicuspids

Piece of bone is removed from mandible and slide everything back

yo

Corrects Prognathism

Corrects Prognathism
Mandibular
o Prognathism

o
o
o

Retrognathism
Asymmetry
Apertognathia

Perform bilateral step-shaped cuts in the body of the mandible

Mandible divided into three independently movable parts


u shouldnt section the entire tooth wile performing a surgical removal of a mandibular molar because there is a
possibility of accidental perforation of the lingual plate and injury to the lingual nerve
Lefort 1 involves maxilla
Facial Fracture Types
Zygomaticomaxillary
Complex

Lefort I

40% of Facial Fractures


Potential dysfunction of the infraorbital nerve
Potential damage to the contents of the superior orbital fissure
Paresthsia
Antrum filled with hematoma
Ocular muscles may be impaired bc fracture of orbital process
Numbness of upper lip, cheek, and nose
Blindness (RARE)

15% of Facial Fractures


Transmaxillary Fracture
Fracture transverse the maxilla through the maxillary sinus along the floor of the
nose
Allow maxillae and hard palate to move separately from upper face

11

Lefort II

10% of Facial Fractures


Pyramidal Fracture
Fracture separates the maxilla and the nasal complex from the orbital and
zygomatic structures
Maxillary and nasal complex move separately from upper face

Lefort III

10% of Facial Fractures


Craniofacial Separation
Allows the entire upper face to move separately from the skull
Rhinorrhea

Zygomatic Arch

10% of Facial Fractures

Alveolar Process of
Maxilla
Smash Fractures
Other

5% of Facial fractures

5% of Facial Fractures
5% of Facial Fractures

Know ASA classifications


American Society of Anesthesiologist Classification
Class 1

Healthy patient, no medical problems


Class 2

Mild Systemic Disease (controlled)


Class 3

Severe Systemic Disease, but not incapacitating


Class 4

Severe Systemic Disease that is a constant threat to life


Class 5

Moribund, not expected to live 24 hours regardless of operation


Class 6

Organ donor
Needed to calculate amount of epi in 1.8 mL carpule of 2% lido w/ 1:100K epi = 0.018 mg.
1% of anesthetic = 10mg/mL
for every 1:100,000 of vasoconstrictor .01mg/mL
max dose of epi is .2mg or 200 g = 11 cartridges
maximum dose of epi for a cardiac risk pt is .04 mg (or 2 cartridges)
for

peds:
4.4 mg/kg is max dose
1 kg = 2.2 lb
1% anesthetic = 10 mg/m;
to calculate max dose:
o obtain patients weight and convert to kg

ex. (44lb child)/ 2.2 lb/kg)= 20 kg


o calculate maximum mg dose

multiply weight in kg by max recommended dosage (kg) x 4.4 mg/kg = mg

(20 kg) x (4.4 mg/kg) = 88 mg


o calculate mg per cartridge (% anesthetic) x (10) x (cartridge size) = mg/cartridge

(2%) x (10) x *1.8) =36 mg/cartridge`

Which technique of anesthesia has the greatest incidence of injecting into a blood vessel?
PSA
Question about normal hematocrit level
The proportion of the blood that is red blood cells is called hematocrit.

12

The hematocrit of adult men averages about 42, while that of women averages about 38
I had a couple questions on when to premedicate a patient
Antibiotics for Prevention of Infective Endocarditis
Adults:

Amoxicillin: 2.0g, 30-60 minutes prior to procedure


Adults allergic to penicillin:
Clindamycin: 600mg, 30-60 minutes prior to procedure
Children:
Amoxicillin: 50mg/kg, 30-60 minutes prior to procedure
Children allergic to penicillin:

Clindamycin: 20mg/kg, 30-60 minutes prior to procedure

Lone maxillary 3rd molar: beware of .. I put tuberosity fracture


The most common complication in removing a freestanding isolated maxillary molar is fracture of the
tuberosity or alveolar process
Maxillary third molars are occasionally displaced into infratemporal space and max sinus
Had a question about preferred order of extractions for molars answer choices mixed order between 1 st, 2nd, 3rd
molars): I picked 3rd, than 2nd, than 1st (beware the lone-standing molar).
The palatal root of the max first molar is the root most often dislodged into the max first molar
o If you have a small oroantral communication, leave it alone and let a blood clot form
o If you have a moderate one (2-6 mm) treat with figure 8 suture over the tooth socket
o If you have a large communication (7mm or more) close with a flap
o Post oroantral communication prescribe the decongestant afrin, the antibiotic amoxicillin, and Actifed, a
systemic decongestant
o Teeth that are impacted into the max sinus are removed via the Caldwell-Luc approach.

The surgical approach to the maxillary sinus is made intraorally with an incision designed to reflect
a flap exposing the anterior wall of the sinus in the canine fossa
If in the extraction of a max third molar you realize the tuberosity was extracted, treat by smoothing sharp edges
of the remaining bone and suture soft tissue. The treatment of a fractured but intact tuberosity during tooth
extraction is to reposition the tuberosity and stabilize with sutures.
Dry socket is most likely cause by : ans choices were physiological, functional, loss of fibrin and something else
The most common complication seen after extracting mandibular molars is dry socket.
A mandibular third molar root tip that disappears is most likely in the submandibular space, but it can also be
dislodged into the mandibular canal or lingual cortical plate.
Treat dry socket by gently flushing out the debris with warm saline solution, then placing a sedative (eugenol)
in the socket (remove the dressing within 48 hours). Prescribe NSAIDS, antibiotics arent needed
Dry socket is a disturbance in wound healing
The Five Phases of Healing of an Extraction Site
1. Hemorrhage and clot formation
2. Organization of the clot by granulation tissue
3. Replacement of granulation tissue by connective tissue and epithelialization of the site
4. Replacement of the connective tissue by fibrillar bone
5. Recontouring of the alveolar bone and bone maturation
A few I&D questions and when to do that and prescribe antibiotics
Criteria for Referring a Patient to an Oral Surgeon for I & D

Rapidly progressive infection

Difficulty in breathing or swallowing

Fascial Space Involvement

Elevated Temperatures (> 101F)

Severe Trismus (<10mm)

Toxic appearance

Compromised host defense


Indications for Antibiotics for I & D

Rapidly Progressive Swelling

Diffuse swelling

Compromised host defense

Involvement of Fascial spaces

Severe Pericornitis

13

Osteomyelitis

For Pharm: A lot of the questions were easy things that were on the tuft doc (not same question but same concept)
They asked me the mechanism of Acetylcholine uptake I think?
Dynamics of neurotransmission
Biosynthetic pathway for ACh

Choline is taken up into nerve via action of permease

Choline acetyltransferase catalyzes the synthesis of Ach from acetyl CoA and choline
Biosynthesis of NE and E

Tyrosine to DOPA (enzyme is tyrosine hydroxylase)

DOPA to Dopamine (enzyme is domain beta hydroxylase)

Mostly in the adrenal medulla: NE to E (enxyme is phenylethanolamine N-methyltransferase)


Tyrosine hydroxylase catalyzes the rate limiting step in the synthesis (this enzyme is inhibited by metyrosine)
The neurotransmitters ACh, NE, and dopamine are stored in vesicles and granules
Termination of transmission by Ach takes place primarily by metabolism by acetylcholinesterase located on
postsynaptic or postjunctional membranes
Schedule 2 drugs cannot be refilled
A lot of the pharm questions were asking the purpose of drugs I have never heard of (like 3-4)- cant even remember
the names bc they were long as shit . the only one I remember is griseofulvin (see chart below)
What do you treat MRSA with? I said Vancomycin
Vancomycin is a narrow spectrum antibiotic (treats gram positive aerobes and anaerobes)
Some sensitive organisms and indications:
1. Staphylococcus aureus (including methicillin resistant staph)
2. Streptococci, enterococci, clostridium difficile
Administration: given intravenously
I had a lot of anti-viral stuff especially for AIDS and herpes
Antifungals and Antivirals
Antivirals Acyclovir
Treats HSV, VZV
Inhibits viral DNA
polymerase after
undergoing
phosphorylation
Amantadine
Used for influenza a
Blocks uncoating of
virus and blocks
replication
Famciclovir,
Iysed for HSV and
Inhibits viral DNA
penciclovir
varicella zoster virus
polymerase after
undergoing
phosphorylation
Foscarnet
Used for HSV and
Inhibits viral DNA
varicella zoster virus
polymerase
Reverse
Used for
Inhibit viral RNA
transcriptase
dependent DNA
inhibitors
polymerase

Antiviral drugs attack the mechanism used by the viruses to replicate and infect. The MOA of most of these
drugs is to inhibit DNA or RNA synthesis and function.
Antifung
Amphotericin
Indicated for most
Amphotericin B
als
B
systemic fungal
produces nephrotoxicity
infections
and hypokalemia as a
Used for serious
side effect
fungal infections
caused by a variety
of fungi
Nystatin

Clotrimazole,
miconaole

Useful primarily for


treating candida
albicans
In the polyene class
Topical antifungal
Useful topically for
candidiasis

Combine with
ergesterol to form
membrane pores

Polyenes cause renal


toxicity, hemolytic
anemia, hypokalemia

The azoles inhibit


ergesterol synthesis

The azoles cause


hormonal imbalance

14

Ketoconazole
Fluconazole
Itraconazole

used systemically for


treating a variety of
fungal infections

Griseofulvin

Inhibits mitosis
Used primarily
against
dermatophytes (skin
fungi)

(especially Ketonazole),
inhibit drug metabolism
(especially
ketoconazole), cause
liver toxicity
Photosensitivity, induces
liver metabolism, liver
toxicity

What organism mostly causes sinusitis in diabetics?


Particularly severe forms of chronic sinusitis are caused by fungi (ex mucormycosis) especially in diabetics
Sulfonylurea stimulates? I said release from pancreas
Sulfonylureas are oral hypoglycemic indicated for type II diabetics(tobutamide, acedohexamide, tolazamide,
chlorpropamide, glyburide, glipizide, glimepiride)
MOA: close K channels in cell membranes, stimulate release of insulin from pancreas, increase sensitivity of
target organs to insulin.
Other oral hypoglycemic: repaglinide (act like sulfonylureas) and metformin (reduces glucose production by
the liver and increases sensitivity to in insulin in muscle, liver and fat cells. These are also used by type II
diabetics
Oral Path: these were mostly things that were in tufts and stuff Dr. Childers embedded in us. They were just pics or a
list of symptoms
I know I got pyogenic granuoloma, osteosarcoma, ameloblastoma, fibroma, multiple myeloma, AOT, stafne, gland
radiolucency in mandible

Bump on the gum differential, pyogenic granuloma, fibroma, peripheral ossifying fibroma, peripheral giant cell
granuloma
o Pyogenic granuloma occurs at any age, most commonly on interdental papilla, inflammatory process,
bleeds readily, exophytic, not painful, grows very fast, proliferative.
o Peripheral giant cell granuloma: looks like pyogenic granuloma, often brownish color, limited to
alveolar ridge gingiva, usually anterior to first molar region
o Fibroma- most common connective tissue tumor, reactive (not a true tumor), common site is tongue
due to trauma
o Peripheral ossifying fibroma: soft tissue lesion, not in bone, but makes osteoid bone. Occurs on
gingiva, especially interdental papilla area
Pyogenic Granuloma (any age)(greater in F)
Elevated mass-Grows fast
Granulation tissue
Ulcerated and bleeds
easily
Most common location:
Interdental papilla

Trauma provides pathway for non-specific


organisms, calculus
Fibroma

Most common Connective Tissue tumor**


Not True tumor

Hyperplasia
Caused by chronic trauma to oral mucosa
Disease
Pregnancy Tumor (Pyogenic Granuloma)

Located on gingiva , lips


and buccal mucosa

Painless
Broad base swelling
Firm/smooth/pink
Found

on
Buccal mucosa,
Lateral border of tongue
Lower lip

Clinical
Elevated mass-Grows

Radiographic

Histopathology
Granulation tissue

15

fast
Ulcerated and bleeds
easily
Located on gingiva , lips
and buccal mucosa
Secondary cause of altered endocrine
system

Osteosarcoma is in the poorly demarcated borders differential

Sunburst appearance
Diseases
Clinical
Osteosarcoma
Mandible:

Swelling/localized pain

Loosening of teeth

Paresthesia due to IAN

Mesenchymal Malignancy that is bone


producing

Maxillary

Swelling/localized pain

Loosening of teeth

Paresthesia to infraorbital
nerve, epistasis, nasal
obstruction and eye
problems

ML radiolucency differential: ameloblastoma, OKC, CGCG


Disease
Clinical
Ameloblastoma
Between 35 and 45
years
Most common in
posterior mandible

Radiographic
Widen PDL space
Moth-eaten RL

Histopathology

Irregular, poorly
marginated
Sun-Ray/sunburst
appearance

Radiographic
Well defined
Multilocular/unilocular RL
Superimposed over posterior
teeth

Histopathology
2 subtypes

Solid

Unicystic

Can cross midline


Slow growing/painless
Causes facial
deformities
Teeth are vital

Most aggressive odontogenic tumor*


Most common epithelial odontogenic
tumor**
Multiple Myeloma

Pain in lumbar or thoracic

Abnormal M spike

Jaws are affected 70%


of cases

Bence Jonce
Proteinuria

Molar-ramus area
Punched out RL in
bone

Anemia

Punched out lesion on Lateral Skull


Radiograph

16

Disease
Stafne Bone Cyst

Clinical
Anatomical depression in
the lingual aspect of the
body of the mandible
where salivary gland
tissue rests

Radiographic
Small circular,
corticated RL below
the level of the
mandibular canal

Histopathology
Normal submandib
gland tissue

Developmental anomaly
Not a true cyst*
The differentials werent bad
Know the differences btw amelogenesis imperfecta, dentinogenesis imperfecta, dentinal dysplasia
They showed x-rays of these (and the x-rays on the exam suck)

Amelogenesis imperfect: teeth lack enamel; dentin and cementum unaffected, shapes of roots and crowns are
normal, enamel is missing,

Dentinogenesis imperfect: opalescent dentin- blue gray, often associated with osteogenesis imperfect (blue sclera,
multiple bone fractures), lack of pulp chambers and root canals, bell shaped crowns with constricted cervical
region

Dentin dysplasia: dentin abnormal with exposure, draining fistulas, misshapen teeth
Amelogenesis Imperfecta
-Enamel abnormality
-Enamel
AR or AD
missing

Hypoplastic
Ectodermal

Hypomaturation
-Normal pulps
Defect

Hypocalcified
and root
canals
-Dentin, cementum, and

Co
Re

pulp unaffected
-Normal shaped tooth
-No increase in caries rate
-Open bite common
Enamel opaque & pitted
Disease
AI (Type I: Hypoplastic)

Normal to opaque white-yellow brown


AI (Type II: Hypomaturation)

Creamy opaque to yellow/brown

Enamel soft and rough

Dental sensitivity
AI (Type III: Hypocalcified)

Opaque white to yellow brown

Enamel soft and rough

Dental sensitivity
Disease
Dentinogenesis Imperfecta

Clinical
-Enamel thin to normal
thickness
-Furrows and pits
-Enamel is normal
thickness but abrades
easily

Radiographic
-Enamel has normal to
slightly reduced contrast
-Enamel has normal to
slightly reduced contrast

Histopathology
AR or AD
Ectodermal Defect
AR or AD
Ectodermal Defect

Open bite common


-Enamel normal thickness
but abrades easily

-Enamel has normal to


slightly reduced contrast

AR or AD
Ectodermal Defect

Open bite common


Heavy calculus
Clinical
-Opalescent dentin
-Gray/bluish-brown

Radiographic
Bell-shaped crown w/
constricted cervical
region

Histopathology
AD

Treatm
Full Co

Mesodermal Defect

-Primary & Permanent


teeth

DI (Type I)

-Blue Sclera

-Obliterated pulps

Full Co

17

-Occurs in patients with


[Osteogenesis Imperfecta]

-Bone fractures
-Primary teeth
affected more

DI (Type II)
Most common Type
DI (Type III)
[Brandywine Type]

-Dentin abnormality

-Obliterated pulps

Full Co

-Dentin abnormality
-Multiple pulp exposures

-Periapical
radiolucencies
-Pulp chambers and
root canals are
extremely large

Full Co

Disease

Clinical

Radiographic

Dentin Dysplasia

-Normal color and


shape

-Short roots

Treatment

-Obliterated
pulp chambers
and canals

-Mobile teeth
-greater resistance
to caries than
normal teeth

Histopatholog
y
AD

-Periapical
radiolucencies

Type I Radicular
Dysplasia
-Primary teeth
=Opalescent
-Permanent
teeth=Normal

Pulp chambersThistle Tube

AD

-Coronal pulp
enlarged Thistle
tube

Type II Coronal Dysplasia

Gorlin has multiple odontomas and okcs, Gardner has multiple osteomas
Gardners- supernumerary teeth GI polyps, osteomas
Gorlins (nevoid basal cell carcinoma)- associated with odontoma, multiple OKC,
I had a lot of symptoms of herpes
Disease
Herpes Simplex Virus 1

(Herpes Labialis)

Clinical
Inflamed gingiva
Any part of the oral mucosa
and lips may be involved
Cold sores
Painful ulcers

Lies dormant in the trigeminal ganglion*


Recurrent herpetic Stomatitis

Triggered by

Trauma

Fatigue

Radiographic

Histopathology
HSV1

Lipschultz Bod

These bodies can


found in a Tzanc
smear

HSV1

Lipschultz Bod

18

Lies dormant in the trigeminal ganglion*


Primary Herpetic Gingivostomatitis

Immunosuppression
Stress
Allergy
Sunlight

These bodies can


found in a Tzanc
smear

Common in young children


Also affect 15-25 y/o
Nearly 90% of all infections are
subclinical(Only have flu-like
symptoms) with one or two
mild sores in mouth that goes
unnoticed by parents

Most serious potential problem is


dehydration

Primary herpes
simplex
Secondary
(recurrent)
herpes simplex
Varicella
Herpes zoster

Virus
HSV1
HSV1
Varicella zoster
virus
Varicella zoster

Location
Perioral, oral,
especially
gingiva,
Lips, hard palate,
and gingiva,

Signs
Vesicles ulcers

Symptoms
Fever mailaise,
painful ulcers

Treatments
Acyclovir,
symptomatic

Vesicles, ulcers

Painful ulcers

Acyclovir

Trunk, head, and


neck
Unilateral trunk,
unilateral oral

Vesicles, ulcers,

Fever, malaise,
painful ulcers,
Painful ulcers

Symptomatic

Vesicles, ulcers

acyclovir

BMMP vs Pemphigus

Desquamative gingivitis refers to epithelium that spontaneously sloughs or can be removed with minor
manipulations

Pemphigoid is primarily mucosal lesions, while pemphigus has vesicles, erosions, and ulcerations on any oral
mucosal or skin surface
Disease
Pemphigus Vulgaris

Clinical
Mucosal erosions
Ulcerations
Nikolskys signepithelium slides off by
rubbing

Radiographic

Histopathology
Antibodies attack
desmosomal adhesio
molecules Dsg3

Acantholoysis
Suprabasilar vesic

Most common form of


Pemphigus
Pemphigoid

Clear-fluid blisters that


breaks rapidly in the
mouth, leaving behind
tender flat ulcer
Gingiva mainly
involved
Slough during brushing
and eating

Antibodies attack
attachment fibrils (Ty
VII collagen)
Subepidermal
vesicles

Nikolskys sign-

19

epithelium slides off by


rubbing
Peds:
Pulpotomy and pulpectomy questions (like class stuff but there werent any board questions that thwaites gave us)
Pulp Tx/Tooth Trauma
When should an apexification procedure indicated? Necrotic pulps with open apexes
What material is typically used to encourage root formation in apexogenesis procedures? MTA
What is the best sign of a successful apexogenesis procedure? Continuous completion of the apex
Pulp Therapy is contraindicated on children with? Serious illnesses (Leukemia, cancer)
Which primary tooth is commonly known for not having accessible canals? Primary molars
Indications for Primary Tooth Endodontics

A tooth that is restorable with a stainless steel crown

No pathological root resorption

Layer of overlying bone between permanent tooth bud and area of pathological bone
resorption

Suppuration

Pathological periapical radiolucency


Contraindications for Primary Tooth Endodontics

Floor of pulp opens into bifurcation

Extensive internal resorption tooth is weakened. cant support stainless steel crown

More than 2/3 the roots have been resorbed

Teeth without accessible canals


PULPOTOMY
Pulpal exposures in primary teeth are indications for what procedure? Pulpotomies
What is the most commonly used medicament for pulpotomies on primary teeth? Buckleys formocresol
When is formocresol indicated? Primary teeth with vital root tips
What procedure is formocresol used in? Pulpotomy
Formocresol consist of? 19% Formaldehyde, 35% cresol, 15% glycerin and water
When doing a formocresol pulpotomy it is important to dry cotton pellets with cotton rolls? True
How long should cotton pellets with formocresol are left on pulp stumps? 5 minutes
What color does tissue turns after formocresol fixation is complete? Brownish-purple
What is the mechanism of formocresol? Tissue fixation and degeneration of odontoblast

After formocresol pellets are removed, ZOE is used to obturate the pulp chamber and tooth is restored? True
A diluted formocresol pulpotomy has been recommended to produce good long-term therapeutic results. What
is its dilution? One-fifth Formocresol dilution/20% solution
What medicaments are potential replacement procedures for formocresol pulpotomies? Glutaraldehyde
Pulpotomy, Ferric Sulfate Pulpotomy, Minerl Trioxide Aggregate
Know which disorders have cleft palate
Syndrome
Aperts Syndrome

Oral manifestations

Class III malocclusion (Cranial sutures fuse too early)

Prominent mandible

High arched palate/cleft palate

Bifid uvula

Crowded maxillary teeth/supernumerary teeth

Shovel-shaped incisors

Crouzon Syndrome

Maxillary hypoplasia
Reduced width of dental arch w/crowded teeth
Calcified stylohyoid ligaments
Unilateral or bilateral posterior crossbite
Spaced eyes and protruding eyeballs

Riegers Syndrome

Hypodontia
Underdeveloped premaxillary area
Cleft palate and protruding lower lip

20

Treacher Collins
Syndrome

Delayed sexual development and hypothyroidism


Cleft palate
Shortened soft palate
Anterior open bite
Enamel hypoplasia

Down syndrome less caries but more periodontitis


Down syndrome has underdevelopment or hypoplasia of the mandible, its associated with class III
malocclusion. Patients often have open bites, tooth eruption is delayed. The tongue is often fissured, and
macroglossia is usually relative to the small oral cavity. Dry cracked lips often result from a protruding tongue
and frequent mouth breathing
Patients with Down syndrome and heart defects often require subacute bacterial endocarditis prophylazis
Acute herpetic gingivostomatitis age range? Under 3 y/o
Puberty Gingivitis
What are the characteristics of puberty gingivitis? Enlarged interdental gingiva and
spontaneous bleeding
What is the treatment for puberty gingivitis? Professional cleaning and improved
oral hygiene
Primary Herpetic
Gingivostomatitis

What causes Primary Herpetic Gingivostomatitis? HSV-1


What age group is primarily affected by Primary Herpetic Gingivostomatitis? Children
under 3 years old
What is the treatment? Rinsing with 50:50 suspension Benadryl Kaopectate
and/or Viscous Lidocaine

Acute Herpetic
Gingovstomatitis

How is Acute Herpetic Gingivostomatitis treated if diagnosed within 3 days of onset ?


Acyclovir suspension [ 15mg/kg 5x a day for 7 days]
How is Acute Herpetic Gingivostomatitis treated if diagnosed after 3 days of onset?
Palliative care [ plaque removal, NSAIDs, topical)

Recurrent
Herpetic Simplex
(herpes labialis)

What type of herpetic lesions are associated with emotional stress? Recurrent
Herpetic Simplex
Where are Recurrent Herpetic Simplex lesions located? Jxn of lips, corners of mouth,
and beneath the nose

Questions on calicification times: showed a pic with hypocalcified teeth and asked when it happened
Errors During Tooth Development
Stage
Time
Consequences
Initiation
6-7 weeks
Anodontia and Supernumerary Teeth
Bud Stage
8 weeks
Cap Stage
9-10 weeks
Dens-in-dente (Dens Invaginatus)
(proliferation)
Germination
Fusion
Tubercle Formation
Bell Stage
11-12 weeks
Macrodontia and Microdontia
(Histodifferentiation and
Peg lateral incisors
Morphodifferentiation)
Dentinogenesis Imperfecta
Amelogenesis Imperfecta
Apposition
Varies per tooth
Enamel Dysplasia
Enamel Hypoplasia
Concrescence
Enamel Pearls
Calcification
Varies per tooth
Hypocalcification Due to infection, trauma or systemic
fluoride ingestion

21

Eruption
Varies per tooth
Attrition
Varies per tooth
Was asked a question about when (what week) dental lamina for succedaneous teeth begins forming. (12 weeks - Bud
stage)
F questions from chart What would you give a 7 year old living in a community with no fluoridation? 1.00 mg/day
Dietary Fluoride Supplement Schedule
Age of Child
Birth -6 months
6months-3years
3-6 years
6 years up to at least
16 years

<0.3ppm F
0
0.25mg
0.50mg
1.00mg

0.3 -0.6ppm F
0
0
0.25mg
0.50mg

>0.6ppm F
0
0
0
0

Whats the maximum amount of F you can safely put in water system
The range is .7 ppm- 1.2ppm
Edge to edge most likely to become class 1
One or 2 questions on sna and snb and what class
Cephalometric measures:
Sna: A-P position of the maxilla: bigger means maxilla is more anterior
SNB: A-P position of the mandible: bigger means mandible is more anterior
Causes of open bite
Open bite in the primary dentition is usually due to habits like thumb sucking and finger sucking
Amelogenis is commonly exhibits open bite
People with down syndrome often have an open bite
People with treacher Collins have an anterior open bite
Bilateral fracture of the condylar neck cause an anterior open bite and an inability to protrude the mandible
*
I think I had one question on space maintenance and it was just like ortho class I had like 4 asked which would be
appropriate for given situations (loss of Primary second molar, loss of Primary central incisor; cant remember the
others)
Space Maintainers
Band and Loop

Prevents mesial movement of second primary molar with the premature loss of
the primary first molar

Space is maintained for developing premolar

Premature loss of First Primary Molar

Distal Shoe

Mandibular Lingual
Arch

Contraindications of a Distal shoe


o Blood Dyscrasia
o Congenital heart defect
o Diabetes
o Immunosuppresion
Prevents posterior teeth from tipping mesially
Prevents lingual movement of incisors following the premature loss of primary
canines
Can be used in permanent dentition when bicuspids are missing and space is
needed for ortho/prosth treatment
Lower Lingual Holding Arch
Premature loss of Primary Mandibular Canine(s)

Premature bilateral loss of maxillary primary teeth

Nance Appliance

First Permanent Molar erupts against distal shoe


Space is maintained for developing premolars
Prevent unerupted first molars from moving mesially with the premature loss of
the second primary molar
Premature loss of Second Primary Molar

22

Has acrylic button that rest on palate

Which one is most like epiphyseal plate? Choices were Synostosis, cranial base, something like that
During adolescence, two competing phenomena occur: the growth rate of long bones accelerates and at the
same time, hormonal changes cause gradual ossification of the epiphyseal plates (syntosis)
Serial ext order: primary canines, then primary molars, then permanent 1 st PM
Prostho:
A picture of epulis fissuratum (see picture on right)
An epulis fissuratum is a hyperplastic tissue reaction caused by ill-fitting or
overextended flange in a denture
Patient comes in wants a new denture, has a complete denture already. You notice
white lesion on mandibular posterior on buccal. What do you do first? Adjust
denture and tell them to come back, do biopsy, excise, or make new denture?
Know combination syndrome
When a patient wears a complete max denture against natural anterior teeth they will experience a loss of bone
structure in the anterior maxillary arch
Palatal extension of maxillary denture
Limiting Structures of the Maxillary Denture
Anterior Region

Labial vestibule
Posterior Region

Junction of movable and immovable tissue

Line is drawn through the hamular notches, 2mm posterior to the fovea palatine
(vibrating line)
I had a ton of questions on adjusting bites, which cusp surfaces to grind, and what to do on articulator with mounted
casts in order to increase vdo
Occlusion
SELECTIVE GRINDING
What is the purpose of selective grinding? To remove all interferences without destroying cusp height
Which cusps are not to be grinded when during selective grinding of artificial teeth? Upper lingual or lower
buccal cusps
Which surfaces should be grinded when attempting to achieve a forward slide from centric relation? Mesial
inclines of maxillary teeth and distal inclines of mandibular teeth
What are the primary holding cusps? Maxillary Lingual Cusps (Never grind)
What are secondary holding cusps? Mandibular Buccal Cusps
When is it ok to grind mandibular buccal cusps? Only if there is a balancing side interference (inner
inclines)
When should cusp tips be grinded? Premature centric, lateral, protrusive contacts
Selective Grinding in Working Side Relations [Rule of B-U-L-L]

Buccal cusp inner inclines of Upper teeth

Lingual cusps inner inclines of Lower teeth

When there is a surface-to-surface contact on flat cusps it should be changed to? Point-to-surface contact

Basic Principles for Occlusal Adjustment


Maximum distribution of occlusal stresses in centric relation
Forces of occlusion (should be borne as much as possible by the long axis of the tooth) (A)
Surface-to-surface contacts on flat cusp , should be changed to point-to-surface contacts (B)
Never take teeth out of Centric Occlusion once its established

23

S sound is when teeth are almost touching


Speech Sounds
Fricative/Labiodental Sounds

Sibilant/Linguoalveolar Sounds

Linguodental Sounds

B,P, and M sounds

F, V, and ph sounds
Maxillary incisors contact the wet/dry lip line of mandibular lip
Determine the position of the incisal edges of the maxillary
S, z, sh, ch, and j
Tip of tongue in most anterior part of palate or lingual surface of
teeth
Help determine vertical overlap of anterior teeth
This, that, those
Tip of tongue protrude slightly between maxillary and mandibular
anterior teeth
Help determine the labiolingual position of anterior teeth
Made by lip contact
Insufficient lip support by the teeth or labial flange can affect these
sounds

Porcelain veneer keeps chipping on incisal edge, whats the cause dont know
The following are contraindications for veneers:
o Patients who exhibit tooth wear as a result of bruxism
o Short teeth
o Teeth with insufficient or inadequate enamel for sufficient retention
o Existing large restorations or endodontically treated teeth with little remaining tooth structures
o Patients with oral habits causing excessive stress on the restoration
Know value, hue I had ~ 3-4 questions on these. Orange stain changes hue
Which is the most important in selecting shade?
Standard Descriptions of Color
Chroma

Saturation of color
Value

Lightness (brightness) or darkness of a color


Hue

Color tone
The single most important factor in shade selection is value
Hue should be selected first when shade selection
When shade selecting examine it under two light sources. Dont select shades under fluorescent light because
fluorescent light is heavier in blue and green wavelengths.
o Select Hue, then Chroma, then Value
o Most important is Value
I had a question about what quality of color is associated with brightness Options were hue, value, chroma,
intensity. I think that its chroma, but Im not positive.
Operative:
5.5 critical pH
Know the pic that Dr. Fryer showed us: base of triangle at dentin

24

Pit and fissure caries: mostly S sanguis and other strep, narrow at the enamel surface and widens at the
DEJ (inverted V. Rapid destruction as many dentinal tubules are involved. Actual lesion is often larger than
clinically presentable. Lesion progression parallels the enamel rods. Prevent with fissurotomy and sealant
Smooth surface caries: interproximal or cervical. The second most prevalent caries. Usually found just
gingival to the proximal contact. Start wide at the surface, and converge towards the DEJ (V shaped)
Dentin caries: dentin has less mineralized tissue and more tubular structures which allows for spread of the
acidogenic destruction (different from enamel) faster progression than enamel caries because there is less
mineral content. V shaped caries with broad base at the DEJ and the apex towards the pulp
Zones of carious enamel: aka zone of incipient lesion. 4 zones have been characterized in a sectional
incipient lesion:
1. Translucent zone deepest zone, named according to its absent or composition less appearance seen
under polarized light
2. Dark zone represents remineralization and is called this because it cant transmit polarized light
3. Body zone largest zone, represents a demineralizing phase
4. Surface zone outermost zone, seems unaffected by the caries

Most common cause of fracture of marginal ridge of amalgam restoration:


Causes of marginal ridge fracture:
o Not rounding axiopulpal line angle
o Marginal ridge too high
o Improper occlusal embrasure form
o Improper removal of matrix band
o Overzealous carving of restoration
A .5 mm ditch in amalgam restoration, how do you treat?
A ditch or gap is the deterioration of the amalgam-tooth interface because of wear, fracture, or improper tooth
preparation
Shallow ditching less than .5 mm deep usually isnt a reason for restoration replacement. The eventual selfsealing property of amalgam allows the restoration to continue serving adequately if it can be satisfactorily
cleaned and maintained. If the ditch is too deep to be cleaned or jeopardizes the integrity of the restoration or
tooth structure, the restoration should be replaced.
To prevent ditching the margins, trim the margins of an amalgam restoration with a sharp instrument
Difference btw 245 and 330 bur
245: has a head length of 3 mm and a tip diameter of .8 mm. The sides are slightly convergent towards the
shank so it creates an occlusal convergence of the facial and lingual prep walls, providing adequate retention
for the prep. It has slightly rounded internal line angles that make the tooth more resistant to fracture from
occlusal force
330: smaller version of the 245, it is indicated for the most conservative amalgam preps
Asked a question about a prep cut with 245 walls are converging occlusally.

25

Patient management:
I had a bunch of questions about cohort studies and that section in Mosbys.
types of studies: epidemiological studies can be organized into three categories
o descriptive: used to quantify disease status in the community. The major parameters of interest are
prevalence and incidence

prevalence indicates what proportion of a population is affected by a condition at a given


point in time. It is expressed as percentages. Ex. The prevalence of periodontal disease
among 100,000 adolescents was 5%

prevalence = Number of people with the disease


total number of people at risk

incidence: indicates the number of new cases that will occur within a population over a
period of time (ex. The incidence of people dying of oral cancer is 10% per year in men aged
55-59 in our community
o incidence= number of new cases of the disease/ total number of people at risk
o analytical epidemiology: used to determine the etiology of a disease. The researcher tries to
establish a causal relationship between factors and disease. Three study designs are used: cross
sectional stud, case control study, and cohort study (prospective and retrospective)

cross sectional study: a study in which the health conditions in a group of people in a cross
section (population) is assess at one time.

Its quick and inexpensive, but the potential to contribute to a judgment of causation is
limited because it cant determine whether the outcome occurred before the behavior
in question, or if it developed because of another cause

Case control study: people with a condition (cases) are compared with people without it
(controls) but who are similar in other characteristics.

Cohort study

Prospective cohort study: a general population is followed through time to see who
develops the disease, and then the various exposure factors that affected the group
are evaluated. Following the group over a period of time, the investigators describe
the prevalence of outcomes

Retrospective cohort study: used to evaluate the effect that a specific exposure has
had on a population. The investigator chooses or defines a sample of the subjects who
had the outcome of interest and looks back at risk factors that may have predicted the
subsequent outcome
o Experimental: compare the incidence of disease and the side effects between the groups in the study
to draw inferences about safety and efficacy of the treatment under investigation

Well designed trials use a double blind design (neither subject nor investigator knows which
group a subject belongs)
Ex. A dentist decides to give out a survey to each patient as they leave his office for one month- what kind of study is
this?
Which type of sterilization uses highest T? dry heat
autoclave -121 degrees Celsius
132 degrees Celsius for a Harvey chemiclave
160 degrees Celsius for dry heat
heat kills microorganisms through protein denaturation
What part of the xray is the dental assistant most likely to get exposure from
Had a question about the most common source of radiation exposure in society choices were, medical-related,
occupational/industrial-related, and two others (I put industrial, but I have no idea)

26

Day 2: random cases , 100 questions total (similar to case presentation class with Dr. Galloucis)
I cant really remember the cases. They give you a med HX, a pan, sometimes FMX, and sometimes pics. The
radiographs are awful.
There will be a list of meds the patient is on and they will expect you to know side effects of these meds, or side
effects of the disorder that the meds are treating.
I had one patient on Chantix (to stop smoking)
I had another on Adderall (for ADHD)
I had a patient with HIV and a obviously on a bunch of meds
I had a child with a little bump on the gums over a central primary incisor with the permanent coming in.
They asked if her PAN coincided with her real age or if one was lagging
They asked where the bump could be coming from (she did fall and fracture the adjacent tooth)
They asked about a space on her pic and if she needed a space maintainer
There was a radiolucency on the medial aspect of a primary 1 st molar and they wanted to know what to do. I couldnt
even see it on the xray
Another case with the child on Adderall, all I remember is them asking if he should be taken off the adderall for the
appointment if he was getting multiple restorations done
There was a patient with a PAN showing a tiny radiolucency over the central incisior. The pic showed a slightly dark
mark on the gingiva that could have been at the same spot. I said it was an amalgam tattoo, however it was right
above an endo-treated tooth and in one radiograph looked like it was gutta percha past the apex. In another Xray it
looked slightly lighter than the gutta percha- I looked at that thing forever trying to figure out if it was the gutta percha
bc my entire exam cast a double shadow (including the wording on the questions so imagine how the radiographs
looked). Then they asked how you would treat the tooth? Leave it alone, retreat it or do surgical retreat.
I had a patient on IV bisphosphonates and chemotherapy on a bunch of wacky drugs asked which one was the IV
bisphosphonate (I put ibandronate aka Boniva. I believe one of the other choices was tiludronate aka Skelid, which
is oral only).
Afurays Mix.
Dont ask me how I got this but this were most of my questions.
Pt overdosed on BDZ, what do you administer..flumanezil
Pt overdosed on narcoticnaloxone
What component is present in IRM that is not present in ZOE IRM is ZOE with PMMA beads
o Irm is a polymer reinforcement zinc oxide eugenol composition
Which is the best area to obtain a free gingival graft
o The palate is the most common donor site for the fgg and CTG. The ideal thickness of the free gingival graft is
1-1.5 mm (page 258 mosbys)
Know what area is most successful for dental implant. I said posterior mandible. Some books say anterior mandible.
Please verify!
Know minimum amount of space between two implants -3 mm between implants and 1 mm from tooth
Also, minimum amount of space between implant and inferior alveolar nerve
o Stay 2.0 mm above the inferior alveolar canal
o Stay 5.0 mm anterior to mental foramen
o And stay 1.0 mm from the PDL of adjacent natural teeth
o Ideally 10 mm of vertical bone dimension and 6 mm of horizontal should be available for implant placement
o Placement at these dimesions prevents encroachment on anatomic structures and allows 1.0 mm of
bone on both the lingual and facial aspects of the implant
TONS of questions on Apexogenesis vs Apexification, chronic periradicular periodontitis and chronic apical
periodontitis!!! Please Know!
(page 3-4 of mosbys)
Apexogenesis
Apexification
Process
Encourage the process of
Stimulate the closure of the
normal root maturation and
apex with hard substance to
closure
allow obturation of the root
canal space
Pulpal Vitality
Vital
Necrotic

27

Acute periradicular periodontitis: painful inflammation around the apex (localized inflammation of the PDL) can
be the result of pulpal disease extending nto the periradicular tissue, canal instrumentation or overfill, occlusal
trauma like bruxism.
o Tooth can be vital or non vital

If tooth is vital, a simple occlusal adjustment will often relieve the pain

If the pulp is necrotic and remains untreated, additional symptoms may appear as the disease
progresses to the next stage- acute apical absecess
Acute periradicular abscess (acute apical abscess): a painful, purulent exudate around the apex
o It is a result of the exacerbation of acute apical periodontitis from a nerotic pulp
o Radiographically may see a normal or slightly thickened lamina dura
o Symptoms:

Rapid onset of swelling, moderate to severe pain, pain with percussion and palpation, slight
increase in tooth mobility,

The acute apical abscess can be differentially diagnosed from the lateral periodontal absecess
with pulp vitality and testing, and sometimes with periodontal probing
Chronic periradicular periodontitits: a long standing, asymptomatic, or mildly symptomatic lesion
o Usually accompanied by raiographically visible apical bone resorption
o Diagnosis is confirmed by:

The general absence of symptoms

The radiographic presence of a periradicular radiolucency

The confirmation of pulpal necrosis


Suppurative periradicular periodontitis (chronic periradicular abscess)
o Associated with a draining sinus tract without discomfort
o The exudate can drain through the gingival sulcus, mimicking a perio lesion with a pocket
o Radiographic examination shows the presence of bone loss at the periradicular area
o Sinus tract resolve spontaneously with nonsurgical endo treatment
Chronic focal sclerosing osteomyelitis (condensing osteitis)
o Excessive bone mineralization around the apex of an asymptomatic vital tooth
o This radiopacity may be caused by low grade pulpal irritation
Also, know protocol if tooth is avulsed >1 hr and <1 hr. Also know about splinting (How long you splint for intrusion,
extrusion and Luxation
Treatment: the first priority is to protect the viability of the PDL
o Immediate implantation improved PDL damage healing prevents root resorption
o If implanting a closed apex that has been avulsed for <60 minutes in storage media

Dont handle the root surface and dont curette the socket

Remove coagulum from socket with saline and examine the socket

Replant slowly with digital pressure

Stabilize with a semirigid splint (for 7-10 days)

Administer systemic antibiotics (penicillin 4x daily for 7 days, or doxy 2x per day for 7 days)
Closed apex that has been avulsed for over 60 minutes
o Remove debris and necrotic PDL
o Remove coagulum from socket with saline, and examine alveolar socket
o Immerse the tooth in a 2.4% sodium fluoride with a pH of 5.5 for 5 minutes
o Replant slowly with slight digital pressure
o Stabilize with a semirigid splint for 7-10 days
o Administer antibiotic (penicillin 4 x per day for 7 days or doxy 2x per day for 7 days at appropriate
dosage)
Open apex <60 minutes
o Place tooth in doxycycline (1mg/20 mL saline)
o Remove coagulum from socket with saline and examine alveolar socket
o Replant slowly with digital pressure, and stabilize with a semirigid (physiologic splint for 7-10 days)
o Administer pen or doxy
Open apex >60 mins
o Replantation not indicated
Endo treatment: 7-10 days postreplantation
o Extraoral time <60 minutes

Closed apex

Endo treatment is initiated at 7-19 days

If endo treatment is delayed or resorption is present, long-term calcium hydroxide


treatment is given before root canal filling

Open apex

Endo treatment should be avoided and signs of revascularization should be checked

28


At the first sign of infected pulp, the apexification procedure is begun
Extraoral time > 60 minutes

Closed apex: the same protocol as <60 minute time

Open apex (if replanted)

If endo treatment wasnt performed out of the mouth, the apexification procedure is
initiated
If there is a combined perio-endo lesion, which would you treat first endo first . Perio therapy should be addressed first
in cases with primary perio lesions
Know which cells predominate in a healthy pulp vs. hyperemic pulp
in a diseased pulp, PMNs plasma cells, basophils, eosinophils, lymphocytes, and mast cells are present
strict anaerobes play a significant role in periapical pathosis
Chronic Cellular

Plasma cells
Response

Macrophages
(No direct exposure)

Lymphocytes
Acute Cellular response

Polymorphonuclear (PMN) Leukocytes


(pulpal exposure)
o

Know the cells dominate in early stages of gingivitisPMNS, lymphocytes, and plasma cells. The answer for that
particular question was plasma cells b/c it asked for that specific time frame (see chart in perio section of Mosbys) see
cat breakdoown
Shape of access opening for mandibular canine
Max central: oval triangular,
Other anterior teeth and premolars: oval
Max molars: triangular with base at buccal cusps
Man molars: trapezoidal
Know sequence of teeth extraction for serial extraction
Primary canines, then primary molars, then permanent 1 st premolar
Know fluoride chart cold see cat breakdown
Know reduction for functional cusps vs. nonfunctional cusps
Never grind maxillary lingual cusps (primary holding cusps)
Secondary holding cusps are mandibular buccal cusps, its ok to grind these if there is balancing side
interferences (inner inclines)
Cusp tips should be grinded in premature centric, lateral, protrusive contacts
Where is porcelain strongercompressive strength (options were tensile, etc)
Porcelain is stronger under compressive forces
If a natural tooth is opposed to porcelain, what is the restoration for the tooth in question
Which of the following materials would give the best result in wear resistance? Gold
Ectodermal dysplasia vs. cleidocranial dysplasia . know about both please!
Ectodermal dysplasia- x linked recessive (affects more males than females), hypertrichosis, anhidrosis,
Anodontia or oligodonitia, cleidocranial dysplasia: supernuperary teeth, hypoplasia of clavicles, r
Source agent for herpanginaCoxsackie virus
Herpangina (stomatitis) is in ulcerative conditions differential, location of ulcerations is in posterior soft palate
and nasopharynx. Patient will have sore throat and difficulty swallowing, mild fever, and last 1 week
Recurrent aphthous ulcer
Recurrent painful ulcers (not preceded by vesicles)
Appear on wet (not vermillion) nonkeratinized oral mucosa (not hard palate or gingiva
Three types: minor, major, herpetiform,
May be seen in association with some systemic diseases
HSV1
Virus
Location
Signs
Symptoms
Treatments
Primary herpes
HSV1
Perioral, oral,
Vesicles ulcers
Fever mailaise,
Acyclovir,
simplex
especially
painful ulcers
symptomatic
gingiva,
Secondary
HSV1
Lips, hard palate,
Vesicles, ulcers
Painful ulcers
Acyclovir
(recurrent)
and gingiva,
herpes simplex
Varicella
Varicella zoster
Trunk, head, and
Vesicles, ulcers,
Fever, malaise,
Symptomatic
virus
neck
painful ulcers,
Herpes zoster
Varicella zoster
Unilateral trunk,
Vesicles, ulcers
Painful ulcers
acyclovir
unilateral oral
Systemic condition

Oral lesions

29

Chrohns disease
Behcets syndrome
Celiac sprue
Aids

Granulomatous inflammation of GI
tract
Immunodysfunction featuring
vasculitis
Gluten sensitive enteropathy
Immunodeficiency

Minor apthae
Minor apthae
Minor apthae
Major apthae

Had picture of Stafne cyst

Stafne defect: a developmental concavity of the lingual cortex of the mandible,


usually in the third molar area, that forms around an accessory lateral lobe of the
subman gland and has the radiographic appearance of a well circumscribed cystic
lesion within the bone, usually below the inferior alveolar canal
Picture of radiolucent lesion between maxillary central incisorsnasopalatine canal

If attempting an extraction of a maxillary third molar and tooth is displaced posteriorly and superiorly, where will it be
locatedinfratemporal space
What is the depth to which brushing goes into the sulcus, what is depth that flossing goes into the sulcus.
Brushing only reaches a depth of 1-2 mm subgingivally
Which medication is the best med to tx systemic fungal infection.-nystatin (topical), ketoconazole (used systemically
for treating a variety of fungal infections) , amphotericin B (indicated for most systemic fungal infections), clotrimazole
(topical)
Amphotericin B is indicated for most systemic fungal infections, ketoconazole, and fluconazole is used
systemically for treating a variety of fungal infections
What receptor do opioids act on to cause their effect ..mu
Opioid receptors
o Mu: largely responsible for mediating euphoria, reduced GI motility, physical dependence, and
respiratory depression

Mu1- analgesia

Mu2 respiratory depression, bradycardia, physical dependence, euphoria


o Delta: analgesia, modulates activity on mu receptor
o Kappa: analgesics, sedation, dysphoria, psychomimetic
Compound
Mu
Delta
Kappa
Morphine
++
+
Fentanyl
+++
++
Pentazocine
Partial agonist
++
Buprenorphine
Partial agonist
Antagonist
Met-enkephalin
++
+++
Beta endorphin
+++
+++
Naloxone
Antagonist
Antagonist
Antagonist
Naltrexone
Antagonist
Antagonist
Antagonist
Each receptor mediates analgesia
+ means agonist, the additional pluses indicate relative potency
What I biggest advantage of using nitrous oxide as sedative You must know this Pg.441
Im guessing anti-anxiety properties
Calculation of amounts of ml of anest., and vasoconstrictor can be given to pts.
Effects of Sjorgen syndrome
dry eyes, dry mouth (sicca)
What med increase salivation ? pilocarpine and cimvemiline
What is disadvantage of using NiFi files compared to stainless steel files = you will not see when the file gets old
Which of the following do not cause gingival enlargementphenytoin (anticonvulsant), cyclosporine
(immunosuppressant), nifedipine(high blood pressure), digoxin (cardiac glycoside) does not cause gingival
enlargement
Define if case is primary perio/endo lesions or primary endo/perio lesion
primary endo lesion: tooth tests nonvital, inflammatory processes may appear along the lateral aspects of the
root or in the furcation, or may have a sinus tract along the PDL appearing like a narrow deep pocket

30

primary perio lesions: perio disease starts in the sulcs and migrates to the apex as deposits of plaque and
calculus produce inflammation that cause loss of surrounding alveolar bone and soft tissue. Broad based
pocket formation, and teeth are vital
primary perio lesion with secondary endo involvement: deep pocket with history of extensive periodontal
disease, possibly past treatment history
Which bacteria is found in normal flora gram positive
Know abfraction lesion
Know what medicare is and what it coverage for dental procedures
What term defines color saturationchroma, hue , value chroma
Which test would you use to analyze proportions of men and women with oral cancer chi square
Bacterial flora of aggressive perio
a.a is the primary etiological agent of aggressive perio
aggressive perio also has p. gingivalis, capnotophaega, spirochetes,
Difference b/w fear and anxiety
fear decreases pain and anxiety increases pain
Difference b/w acute periapical abscess and acute periodontal abscess already covered
Pic of zygomatic process see cats breakdown
Pic of papilloma see cats breakdown
Pic of intermaxillary suture

What herbal supplement strengthen the effect of antioxidants.chamomille, st. worts.


st johns wort inhibits the reuptake of serotonin at neuronal synapses resulting in the elevation of seraotnin
within the CNS. This is similar to antidepressants
chamomile: helps digestive ailments
St. johns wort acts as whatantidepressant
What does an area of implant need to have, mm of space buccal lingually = 1 mm on each side making the bone
dimention bucal/lingually at least 6mm wide
Pt. on ginsing what do u want to avoid? Warfarin/nsaids
When would u tx an avulsed tooth with calcium hydroxide therapy related to splinting2 wks after splinting,
immediately before
splinting, after splinting and evaluation
10 days to 2 weeks after replantation, the root canal is prepared(cleaned and shaped) and calcium hydroxide
past is plaed into the canals. This paste is replaced every 3 months for one year. After one year, t appears
that resotption has reversed ot stopped, a permanent gutta percha can be placed. I
f a tooth is out of the mouth for more than 2 hours, ankylosis and external root resortption (most common
cause of reimplant failure) can occur in 2 years rct is perfomed in its entirety prior to replantation. Rinse tooth
with saline, replant into socket and splint for a max of 2 weeks. Resorption is the most frequent sequel to
replantation
How would use remove maxillary torusdouble y incision
How remove mandibular tori txd area if tissue over area becomes denuded and sloughs off
Conventional osteotomies vs. distraction osteotomy procedures
Alveoloplasty indications
surgical prep of the alveolar ridges (removing undercuts and sharp edges from areas like the mylohyoid ridge
for the reception of dentures or shaping and smoothing the socket margins agter extractins of teeth with
subsequent suturing to insure optimal healing
What xray best for determining midface fractures CT
What do you do with endo tx tooth with lateral canal w no material in the lateral canal.retreatement, wait and
evaluate later
What caused pain in pt with previous RCT overinstrumentation of canal, separated file w/in canal, overextension
apically, breakage of apical seal

31

Pic of vertical root fracture - j shaped lesion

Pic of MRI asked what image was


How does gingival tissue connect to implant
implants have a biological width of 3-4 mm.
ct fibers are present as circular fibers oriented parallel to the implant or abutment surface, but no connective
fibers insert into the titanium
osseointegration means direct bone to implant contact with no intervening periodontal ligament or any other
tissue. There is no insertion of the ct finbers into titanium.
Within a few weeks of placement of the implant, woven bone is laid down at the bone implant interface.
Woven bone is characterized by a random orientation of fibrils. Within a few more weeks, the woven bone
becomes lamellar bone. The conversion to lamellar bone is thought to be encouraged by the presentce of
functional forces placed on the implant to stimulate bone
How do you check probing depth of implant
when determining the attachment level on teeth, the cej is used as the landbark to determine the attachment
level.
Because an implant and its restoration have no cej, the shoulder of the implant, or the restoration is used to
tell the relative attachment level instead of the clinical attachment level on a tooth. Probing depths
around implants are determined with a standard shaped perio probe with plastic probes generally
recommended instead of metal probes
What instruments do you use to scale an implant plastic tipped
Cancer from body metastasizes to where in the oral cavity most frequently
Attached gingiva is themost common site
Second most common jaw bone
3rd oral mucosa
What cancer of the oral environment is most malignant mucoepidermoid carcinoma
What is disulfiram used fortx of alcohol abuse
Benzodiazepine MOA
Modulate the activity of the inhibitory neurotransmitter, GABA (increase GABA)
Phenothiazines should not be used with what med (used to treat mental and emotional disorders)
Alcohol: additive CNS depressant effects
Amphetamines: may cause exacerbation of psychotic symptoms
Antidepressants TCAs= may result in increased chlorpromazine concentration, monitor for adverse effects
Cns depressants: chlorpromazine and other CNS depressants (alcohol antihistamines, general anesthetics,
opiates, or other narcotic analgesics, barbiturates, benzodiazepines and other sedative/hypnotic agents) may
result in additive CNS depressant effects. Monitor to avoid excessive sedation or respiratory depression
Levodopa: phenothiazines may inhibit the antiparkinsonian effects of levodopa due to their dopamine blocking
effects in the CNS.
Epinephrine: patients on chlorpromazine who are hypotensive shouldnt be given epi. Chlorpromazine blocks
peripheral alpha adrenergic receptors, thereby inhibiting alpha agonist effects of epi like vasoconstriction and
increased bp/
Phenothiazines MOA
Block dopaminergic sites in the brain.
Maoi cant be used with with indirect acting sympathomimetics likeamphetamine
Asthma
Tx for asthma/medication: for acute attack-> Aminophylline You must know this Pg.442
if patient has asthma, dont give them nsaids.
Chronic asthma can be treated with steroids
Acute asthma: treat with epi, levalbuteral, albuterol, salmeterol, metaproterenol (or any beta 2 adrenergic
agonist)
- Beta 2 adrenergic agonist: Albuterol, metaproteerol, salmeterol(slow acting) (MAO: beta2 agonist..relax smooth
muscle in lungs rapid sideffect tachycardia and tremor)

32

- inhaled glucorticoids: beclomethasone, budesonide, fluisolide, fluticasone (MAO: increases lipomdulin which
inhibiting phospholipase A2 and Cox2),
- antimuscarinic: ipratropium MAO: blocks muscarinic receptors in lung leading to bronchodilation side effect
xerostomia
- leukotriene synthesis inhibitor: zileuron taken orally reduces inflammation
- luekotriene receptor antagonist: montelukast, zafirlukast (block leukotriene receptor cys-LT1)..long acting
- box 13-3 Management of acute asthma
Drugs: -adrenergic agonists (epinephrine or albuterol) via aerosol, O2, and isoproterenol and glucocorticosteroids
(via an IV route) are used to manage severe acute attacks. You must know this Pg.443 (Malamed, Stanley F.. Medical
Emergencies in the Dental Office, 5th Edition. C.V. Mosby, 012000. 13.5.2).
Syncope most common medical ER in dental clinic caused by #1 vasodepressor due to stress (usually due to
injection) #2 ASA type 3 and 4 patients dont deal with stress well #3drug overdose (opiods, benzos, allergy to
anesth)
o SIMPLE P-A-B-C-D (position, airway, breathing, circulation definitive care
o Step 1: Shake and shout
o Step 2: terminate dental procedure
o Step 3: summon help
o Step 4: position patient head and thorax in same plane and with feet slightly elevated
o 5: Identify if airway obstruction head-tilt chin
- box 5-3 Management of unconscious patients
- (Malamed, Stanley F.. Medical Emergencies in the Dental Office, 5th Edition. C.V. Mosby, 012000.).

tx: treleemberg position, ammonia capsule,


Pregnant woman in supine position? Abdominal aorta, IVC (inferior vena cava), placenta.
Pressure is placed on the IVC
You must know this Pg.444

- Exceptions to the supine position rule include pregnant patients or those with respiratory difficulties and/or chest
pain. A pregnant woman can be placed on her side with the legs slightly elevated to prevent further problems caused
by the weight of the fetus on the vena cava
What causes respiratory problems in children? Asthma?? (can tx: upper resp. problems w/macrolides i.e. Azithromycin,
clarithromycin, erythromycin
Extrinsic asthma, also known as allergic asthma, accounts for 50% of asthmatics and occurs more often in
children and younger adults. Most patients with this form of asthma demonstrate an inherited allergic
predisposition
(Malamed, Stanley F.. Medical Emergencies in the Dental Office, 5th Edition. C.V. Mosby,
Cavernous sinus thrombosis (from abscess of the upper lip)
Canine space and deep temporal space infection can lead to cavernous sinus thrombosis by
the spread of infection via the ophthalmic vein *** Lateral pharyngeal infections (b/w
m.pterygoid muscle and s. phargeal constrictor muscle) can transverse the retropharyngeal and
prevertebral spaces and spread into the mediastinum.
Formation of a blood clot within the cavernous sinus
This area at the base of the brain drains deoxygenationed blood from the brain back to the heart. The
cause is usually from a spreading of infection in the sinuses, ears, or teeth.
S. aureus and Strep are the associated bacteria
Life threatening condition and requires immediate treatment
Vascular congestion (sclera, retina), periorbital edema, proptosis, ptosis, dilated pupils, absent corneal
reflex, and thrombosis of retinal veins. Nerves involved 3, 4, V1, 6
Infective endocarditis in mitral valve/or lesion of upper lip
Prophy dosages ADULTCHILDREN

Amoxicillin 2g 1 hr before.50mg/kg 1hr

Clindamycin 600mg 1 hr before20mg/kg 1hr

Cephalexin or cefadroxil 2g 1hr before50mg/kg 1hr

Clarithromycin/azithromycin 500mg 1hr before..15mg/kg 1hr

Ampicillin 2g IM or IV 30min before50mg/kg 30min

Clindamycin 600mg IV 30 min before20mg/kg 30min

Cefazolin 1g IM or IV 30 min before25mg/kg 30


Congenitally missing tooth BUD (initiation stage) or Cap (proliferation stage)
Initiation phase- anodontia and supernumerary teeth

33

Cap stage period of proliferation, differentiation,a nd morphogenesisdens in dente


Bell stage problems of Histodifferentiation and morphodifferentiation- amelogenesis imperfecta, Macrodontia,
Microdontia
Apposition- cells begin to deposit dental tissues (enamel, dentin, cementum)- enamel dysplasia, enamel
hypoplasia, concrescence
Referred pain to the ear (mandibular molars) You must know this Pg.445
Question (tricky) aversive conditioning/positive punishment ---- know all definitions of these!
Cohort, etc.
??Aversive conditioning positive punishment for doing the wrong habit eg. Adding extra chores in
hopes to decrease neglect in brusing teeth)
Positive reinforcement positive consequence that increases desired behavior (to reward for doing the
right thing)
Cohort
- Retrospective Cohort evaluate the effect that a specific exposure has had on a population (i.e.
Occupational hazard) the investigator chooses or defines a samplelook at a group/population at one
specific time period in the past
- Prospective Cohort to follow a group/population through a time period and evaluate risk factors by
means of surveys to help predict/detect the outcome of interest
types of studies: epidemiological studies can be organized into three categories
o descriptive: used to quantify disease status in the community. The major parameters of interest are
prevalence and incidence

prevalence indicates what proportion of a population is affected by a condition at a given


point in time. It is expressed as percentages. Ex. The prevalence of periodontal disease
among 100,000 adolescents was 5%

prevalence = Number of people with the disease


total number of people at risk

incidence: indicates the number of new cases that will occur within a population over a
period of time (ex. The incidence of people dying of oral cancer is 10% per year in men aged
55-59 in our community
o incidence= number of new cases of the disease/ total number of people at risk
o analytical epidemiology: used to determine the etiology of a disease. The researcher tries to
establish a causal relationship between factors and disease. Three study designs are used: cross
sectional stud, case control study, and cohort study (prospective and retrospective)

cross sectional study: a study in which the health conditions in a group of people in a cross
section (population) is assess at one time.

Its quick and inexpensive, but the potential to contribute to a judgment of causation is
limited because it cant determine whether the outcome occurred before the behavior
in question, or if it developed because of another cause

Case control study: people with a condition (cases) are compared with people without it
(controls) but who are similar in other characteristics.

Cohort study

Prospective cohort study: a general population is followed through time to see who
develops the disease, and then the various exposure factors that affected the group
are evaluated. Following the group over a period of time, the investigators describe
the prevalence of outcomes

Retrospective cohort study: used to evaluate the effect that a specific exposure has
had on a population. The investigator chooses or defines a sample of the subjects who
had the outcome of interest and looks back at risk factors that may have predicted the
subsequent outcome
o Experimental: compare the incidence of disease and the side effects between the groups in the study
to draw inferences about safety and efficacy of the treatment under investigation

Well designed trials use a double blind design (neither subject nor investigator knows which
group a subject belongs)
What % of Fl in water in US? 67% of water in the US with Fl. (0.7-1.2 ppm)
Sensitivity the percentage of persons with the disease who are classified as having the disease.
- True Positive
- Sensitivity = ((TP/(TP+FN)) x100%
Specificity the percentage of persons without the disease who are correctly classified as not having the disease
- True Negative
- Specificity = ((TN/TN+FP)) x100% (percentage)
o True Positive (TP) = Those who have the disease and are correctly identified as having the disease

34

o
o
o
o
o
o
o
o

False Negative (FN) = those who are incorrectly classified as NOT having the disease MISSED
DIAGNOSIS
True Negative (TN) = those who do not have the disease and are correctly identified as not having
the disease
False Positive (FP) = those who have the disease who are NOT identified by the result
Sensitivity vs. Specificity
Sensitivity is defined as the percent of persons WITH the disease who are CORRECTLY CLASSIFIED
as having the disease (those who have the disease)
Sensitivity = TP/(TP + FN) x 100%
Specificity = defined as the percent of people WITHOUT the disease who are CORRECTLY
CLASSIFIED as NOT having the disease (those who do not have the disease)
Specificity = TN/(TN + FP) x 100%

*** These two values are directly inverse to each other as one goes up the other goes down
What population will have recurrent decay? Black, white, Hispanics, native Americans
- Caucasians had mean coronal DFS (decayed filled surfaces) twice as high as African Americans
- Proportion of population that has untreated coronal caries for entire population is three times as higher
in African Americans than Caucasians
- Root caries is the same b/w both populations
Early child hood caries are more common in Latinos
Coronal caries in adults are higher in Caucasians
Untreated caries are three times higher in African American adults than in Caucasians
Waters view for max sinus
Reverse Townes for condylar neck fractures
Oblique view- for position of 3rd molars
Complete dentures. What do they complain about? Lower?
- Possibly dislodgement due to overextension of denture in area of masseter muscle. Or it could be pain due to the
impingement of the denture on the mental nerve/incisal nerve.
- If discomfort in distal lingual s. pharyngeal constrictor muscle is being irritated
You must know this Pg.446
Extending the denture too far posteriorly impinges on superior constrictor. Sore throat when swallowing
o The distolingual extension is limited by the actions of the superior constrictor muscle
An over extended db corner of the mandibular denture will push against the masseter and cause dislodgement
Proper extension into the buccal vestibule provides the best support for the mandibular denture
The Retromolar pad marks the distal termination of edentulous ridge, and this must be covered for support and
retention
Fentanyl is an opioid like morphene. what is used to reverse it? Naloxone, naltrexone (opioid antagonist)

Fentanyl ROA = transdermal.


Barb and benzo overdose= tx: flumazenil
Atropine overdose= tx: pheostygmine
Non-working: lingual incline of mandible facial cusps
Selective grinfing n working side relation. The rule of bull
o Buccal cusp inner inclines of Upper teeth
o Lingual cusp inner inclines of Lower teeth
In balancing side relation:
o Grind the inner inclines of the mandibular buccal cusps (if there is a balancing side interference)
o Never grind the maxillary lingual cusps (primary centric holding cusps)
Know what to give patient for bilateral angular cheilitis
Angular cheilitis occurs in people who habitually lick their lips and deposit small amounts of saliva in the
commissures of their lips, also associated with nutritional deficiencies (Vitamin b2, b3, b6, or b12,
Cam result from fungal candida infections, or from a bacterial (staph) exception
Nystatin will eliminate the fungal infection, and an antibacterial is used to treat the bacterial infection present
- Since candidiasis is usual cause

Nystatin combined w/triamcinolone acetonide cream.topically for angular chelitis w/o a bacterial
component

Clotrimazole cream is useful for angular chelitis w/bacterial component (60% of angular chelitis
is caused by candiadsis + Staph. Aureus)

Angular chelitis has a malignant potential can go to SCC


Mandib denture, the lingual flange, what muscles would affect it? Superior constrictor

35

The dB extension is determined by the position and action of the masseter muscle
The distolingual extension is limited by the action of the superior constrictor
Buccal vestibule is influenced by the buccinators muscle which has muscle fibers that run in an oblique
direction and therefore have little displacing actions. Proper extension into this area provides the best support
for the mandibular denture. This area is also referred to as the buccal shelf
Masseter area: the denture is limited in a lateral direction by the action of the masseter muscle
Retromolar pad: marks the distal termination of edentulous ridge. This structure needs to be covered for
support and retention.
What muscle will the denture sit on? Buccinators
Nitrates for angina. Know how they work.
NO is released causing direct vasodilation.
Nitroglycerin increases oxygen supply to the heart by a direct vasodilatory action on the smooth muscle in
coronary arteries
Also can use ca blockers (verapamil, nifedipine) and propanol to treat angina
Sialoliths stone in the salivary ducts most common site is Submandibular Tx: manually palpate/milk, give
sialologues, excise surgically
Hard mass , high occurrence in the submandibular gland, may be associated with tender swelling of affected
gland. Pain intensifies at mealtime. Treat with surgical extirpation of the sialolith
Mucocele/definition
Mucocele (aka mucus extravasation phenomenon; mucus escape reaction) s a common lesion of the oral
mucosa that results from rupture of the salivary gland duct and spillage of mucin into the surrounding soft
tissues. Due to local trauma. Not a true cyst
Most common sites: lower lip, buccal mucosa, Retromolar region
Recurring submucosal nodule of saliva. Rupture of salivary gland duct and spillage of mucin into the surrounding
tissue usually local trauma but sometimes not.; mucous extravasation phenomenon. Most common site is lower
lip histo: foamy histocytes/macrophages (see the same in Ranula)

Ranula basically a larger mucocele but found in floor of mouth; mucin rupture in sublingual or submandibular
gland tx: extravasation of feeding gland or marsupilazation
Dry socket
Alveolar osteitis
Most common mandibular post extraction complication
- Delayed healing not associated with an infection; primary complication is moderate to severe pain without the usual
signs and symptoms of infection such as fever, swelling, and erythema.

Had a question about which condition would most affect healing time after endo AIDS, leukemia, diabetes,
and one other choice (answer?)
- The term dry socket describes
Dry socket is delayed healing, but is not associated with an infection. Pain develops around the 3 rd or 4th day
after the removal of the tooth. Almost all dry sockets occur after the removal of lower molars. On
examination, the socket appears to be empty with a partially or completely lost blood clot, and some bony
surfaces of the socket are exposed. The exposed bone is sensitive and is the source of the pain. The cause
of dry socket isnt absolutely clear, but it appears to result from high levels of fibrinolytic activity in and around
the tooth extraction socket. This fibrinolytic activity results in the lysis of the blood clot and subsequent
exposure of the bone
Tx includes: irrigation and insertion of a medicated dressing.
Treatment for Dry Socket

Gently flush out debris with warm saline solution

Place sedative (eugenol) in socket (Remove dressing within 48 hours)


o Repeat until patient is asymptomatic

NSAIDs are prescribed if necessary

Antibiotics are generally not indicated***


o Irrigated with sterile saline. Then the socket is carefully suctioned of all excess saline, and small strip of iodoform
gauze soaked with the medication is inserted into the socket. The medication contains eugenol.
o The dressing is changed every other day for the next 3 to 6 days, depending on the severity of the pain.
Definition of allograft
- Graft from another member of the space species DFDBA, FDBA
Materials Used to Correct Osseous Defects

36

Autogenous Grafts

Allograft
Xenograft
Nonbone Graft

Osseous Coagulum
Bone Blend
Cancelous Bone Marrow Transplant
Iliac Cancellous Bone Marrow (Extraorally)
Undecalcified Freeze-Dried Bone Allograft
Decalcified Freeze-Dried Bone Allograft
Bio-Oss (Bovine-derived bone) COW
Bioactive glass: PerioGlas and BioGran
Coal-derived Materials

Malocclusion is least common? Class III (1-2%) Class II (13-25%) Class 1 most common
Prevalence of cleft lip and cleft palate 1 in 700 births (in ortho section of Mosbys) but varies with racein ODR
section of Mosbys, says Cleft lip: 1 in 1000 births Cleft palate: 1 in 2000
- Rule of 10: Tx: cleft lip is done in 10 Weeks, when baby weighs 10 lbs, Hemoglobin is 10g/deciliter
- Epidemiology
o 1/700 overall incidence for facial clefting (not rare)
o Cleft lip +/- cleft palate (CL/P) clusters in families distinct from isolated cleft palate (CP) (different embryology
-- see below)
o Clefting more common in Asians (1/400) and less common in African American (1/2000)
o Clefts can be unilateral or bilateral; Left side more common for unilateral
o Syndromic clefting (pattern of multiple anomalies) accounts for 50-60% pts
o Of these, half are known patterns; others simply show multiple anomalies
o High incidence of congenital heart disease and renal disease -- screen carefully for these
o Other associated midline abnomalities -- hypopituitarism possible
Embryology
o Weeks 4-6: Maxillary processes grow medially & fuse with frontonasal process
o Failure here >> cleft lip +/- primary (anterior) palate
o Weeks 6-7: Tongue descent, migration & fusion of palatal shelves
o Failure here>> cleft secondary (posterior) palate (Pierre-Robin, & other)
o Etiologies
o Teratogens: ethanol (FAS), anti-convulsants, steroids, chemo, excess Vita A
o Maternal / intra-uterine conditions: infant of diabetic mom, amniotic bands
o Chromosomal abnormalities, monogenic causes (AR, AD, XL)
o You must know this Pg.448
o Unknown
Prevalence of cleft palate 1 in 2000 births
o Often have hearing problems and speech problems
o Tx: done around 1 year before speech begins
Presentation had a question about skeletal pattern of pt with Class III Cleft palate: Choices were Mx protruded, Mnd
protruded, Mx retruded, Mnd retruded. (Answer = these pts are Skeletal Class III, but the abnormality is associated
with the maxilla, so Id guess Mx retrusion)
Differentiate cleidocranial dysplasia and ectodermal dys. (I had a lot of questions on these)
o Cleidocranial dysplasia (CCD) absence of clavicles, supernumerary teeth, retained primary teeth; permanent
teeth not erupting, frontal bossing, hypertelorism.
o Autosomal Dominant problem w/chromosome#6.. gene core binding factor alpha 1 (CBFA-1)
o Many supernumerary teeth (so does Gardners syndrome but not as much as CCD
Ectodermal dysplasia abnormalities of two or more ectodermal structures such as hair, teeth, nails, sweat glands,
etc. these people have thin hair, thick nails, lightly pigmented skin, sweat glands that function abnormally (these
people cannot perspire or regulate body temperature); teeth are congenitally absent
o X-linked hypohidrotic ectodermal dysplasia (most well known form)
o Sparse hair, little yey brow hair, light pigmentation
o Oligodontia most common usually not anodontia mutation with Chromosome 14 pax 9 gene

37

Sometimes teeth exhibit taurodontism

Taurodontism:
-

This conditions may exist as an isolated trait (autosomal dominant) or


as part of several syndromes including the trichodentoosseous
syndrome (TDO), otodental dysplasia, ectodermal dysplasia, tooth and
nail syndrome, amelogenesis imperfecta and others
bull like teeth, a molar with an elongated crown and apically placed
furcations of the roots resulting in enlarged rectangular coronal pulpal
chamber elongated pulp chamber, short root canals
Be able to identify radiographically

If Pagets disease of bone (osteitis deformans) occurs in the Jaw will see HYPERCEMENTOSIS
also see hyper cementosis in gardners and acromegaly
Know about dentigonesis imperfecta
Know about Amelogenesis imperfecta:
autosomal dominant condition affecting both deciduous and permanent teeth. Affected teeth are gray to
yellow brown and have broad crowns with constriction of the cervical area resulting in a tulip shape.
Amelogenesis imperfecta: teeth lack enamel; dentin and cementum unaffected, shapes of roots and crowns
are normal, enamel is missing,
Dentinogenesis imperfect: opalescent dentin- blue gray, often associated with osteogenesis imperfect (blue
sclera, multiple bone fractures), lack of pulp chambers and root canals, bell shaped crowns with constricted
cervical region
Dentin dysplasia: dentin abnormal with exposure, draining fistulas, misshapen teeth
Fluoride and the ages
Birth-6months, < 0.3ppm, no fluoride supplementation
6 months 3 years, if < 0.3 ppm then give 0.25 mg; if between 0.3 ppm 0.6 ppm none, 0.6 none
3-6 years, if < 0.3 ppm then give 0.50 mg; if between 0.3 ppm 0.6 ppm give 0.25, 0.6 none
6-16 years, if < 0.3 ppm then give 1.0 mg; if between 0.3ppm 0.6 ppm give 0.50, 0.6 none
Ludwigs angina bilateral swelling causes airway obstruction = this is massive swealing cause by odontogenic
infection
Bilateral swelling of submandibular, submental, and sublingual spaces.
Pic showing either an aneurismal bone cyst or traumatic bone cyst (post mandible)

Amelogenesis imperfect: teeth lack enamel; dentin and cementum unaffected, shapes of roots and crowns are
normal, enamel is missing,

Dentinogenesis imperfect: opalescent dentin- blue gray, often associated with osteogenesis imperfect (blue sclera,
multiple bone fractures), lack of pulp chambers and root canals, bell shaped crowns with constricted cervical
region

Dentin dysplasia: dentin abnormal with exposure, draining fistulas, misshapen teeth
Know about Hunter and Hurlers syndrome. Check Dr Childers lecture.
Hurler: ER, appears in infancy; cloudy corneas, growth retardation, reduced intelligence, coronary artery
disease, rarely live 10 years
Hunter: X linked Recessive. Appears at 1-2 years; clear corneas, reduced intelligence, growth retardation, stiff
joints
These are mucopolysaccharidosis syndromes
Know about pemphigus Vulgaris, lichen planus, cicatric pemphigoid, SLE, Sjorgens syndrome. All in Dr Childers
Lecture.

Desquamative gingivitis refers to epithelium that spontaneously sloughs or can be removed with minor
manipulations

Mucous membrane Pemphigoid (cicatricial pemphigoid; benign mucous membrane pemphigoid) is primarily
mucosal lesions, while pemphigus has vesicles, erosions, and ulcerations on any oral mucosal or skin surface
(explained more extensively in cats breakdown)

Systemic lupus erythematous: autoimmune, young adult females; butterfly rash of face (sun exposure worsens
it). Systemic involvement complications (heart= endocarditis, kidney = renal glomeruli (glomerul-nephritis)

Sjogren: autoimmune disease; not infectious (ex herpes), elderly women. Dry eyes, dry mouth (sicca); parotid
swelling; often other autoimmune diseases (associated with lupus and rheumatoid arthritis)
Nasolabial cyst doesnt involve the bone
Mucolabial smooth swelling adjacent to a max lateral incisor. Superficial soft tissue of upper lip (extraosseous)
- Anogenital condyloma acuminatum, multiple papillary or sessile focal areas of epithelial hyperplasia of the genital
and oral mucosa that contain koilocytes, HPV6, or HPV 11, and is difficult to eradicate strongly linked with HPVs 6 and
11 are probably sexually transmitted.

38

Signs for HPV in oral cavity, oral HPV type 16:


HPV 16 has been identified in some oral leukoplakias
These are the same HPV subtypes associated with uterine cervical carcinoma and a subset of oral squamous
cell carcinomas
HPV-16 has been shown to induce dysplasia like changes in normally differentiating squamous epithelium in an
otherwise sterile in vitro environment
OKC/basal cell nevus aka Gorlin syndrome
Gardners- supernumerary teeth GI polyps, osteomas, odontoma
Gorlins (nevoid basal cell carcinoma)- associated multiple OKC, falx cerebri, bifid rib, frontal bossing, hypertelorism,
palmar and plantar dyskeratosis
Patient were to get a crown, but they want bleaching. Whats the sequence?
o Bleaching is always 1st step and then the restoration is matched to the lightest shade.
Tx sequence
o Oral signs of Addisons disease (hypoadrenocorticism low adrenal corticosteroids b/c destruction of
adrenal cortex) diffuse melanin pigmentation in floor of mouth/ventral surface of tongue (usually
1st sign of Addisons later the bronzing of skin can occur usually in sun-exposed areas)
After flap surgery, how does the tissue heal? Long junctional epithelium
Healing after Flap Surgery
Immediately after suturing (up to 24 hours), a connection between the flap and the tooth or bone surface is
established by a blood clot, which consists of a fibrin reticulum with many polymorphonuclear leukocytes,
erythrocytes, debris of injured cells, and capillaries at the edge of the wound. 3 Bacteria and an exudate or
transudate also result from tissue injury.
One to 3 days after flap surgery, the space between the flap and the tooth or bone is thinner and epithelial cells
migrate over the border of the flap, usually contacting the tooth at this time. When the flap is closely adapted to
the alveolar process, there is minimal inflammatory response. 3
One week after surgery, an epithelial attachment to the root has been established by means of hemidesmosomes
and a basal lamina. The blood clot is replaced by granulation tissue derived from the gingival connective tissue,
the bone marrow, and the periodontal ligament.
Two weeks after surgery, collagen fibers begin to appear parallel to the tooth surface. 3 Union of the flap to the
tooth is still weak because of the presence of immature collagen fibers, although the clinical aspect may be almost
normal.
One month after surgery, a fully epithelialized gingival crevice with a well-defined epithelial attachment is present.
There is a beginning functional arrangement of the supracrestal fibers.

Ideally after reconstructive periodontal surgery you want new attachment with periodontal regeneration
because it results in obliteration of the pocket and reconstruction of the periodontium. However, other
therapeutic results can be seen like :
o Healing with LJW which can result even if filling of bone has occurred
o Ankylosis of bone and tooth with resultant root resorption
o Recession
o Recurrence of the pocket,
o Any combo of these results

- Only in GTR does long junctional epithelium not occur and is by the actual movement of osteocytes movement from
PDL to area
In what order do you extract the molars?
Serial Extractions:
o 1st: primary Lateral incisor (as perm. Erupt only if nec)
o 2nd:primary canine (as perm. Lat. Erupt).. 8-9 yrs
o 3rd: primary 1st molar (6-12mos. Before normal exfoliation).. done to erupt 1st PM to erupt before normal
time so they can be extracted.. and permit Canine to move distally into space 9-10 yrs
o 4th:perm. 1st PM (just as canine emerges through mucosa
Max. canine. Know if they have two canals

39

Max. canine only has 1 canal mandibular canine can have 2 canals 30%

Canine frenum attachment

High renum attachment can cause localized recession in kids


Local Anest. Patient has an adverse reaction to it.
o Probably due to preservative methylparaben (1mg/ml)
o Prilocaine metabolie o-toludine can cause methohemoglobiniea
Amides (lidocaine, Mepivicaine, bupivacaine, Prilocaine) are metabolized by the liver, have few allergies
than esters (procaine, Tetracaine, cocaine) esters are metabolized by esterases in plasma
Prilocaine causes methemoglobunimeia
Local anesthethics cause CNS system effects like: lightheadedness, dizziness, muscle twitching,
convulsions, and cardiac effects like cardiac depression, specific antiarrhythmic effects, and cardiovascular
collapse due to myocardial depression, and hypotensive shock
Kennedy class I, no retention? Where is the problem?
Class 1 is strict tissue retained problem could be in base?
An unseating occurs in the edentulous segments, a line through the rests located furthest from the retentive
clasp tips acts as the fulcrum in a class III lever system. Moving the fulcrum line still further from the clasp tips
improves the mechanical advantages of the lever arm system. By maintaining this position, the most distant
rests augment the retentive action of the clasp and indirectly contribute to retention. Rests = indirect
retainers, which augment mechanical retention
o As the denture base moves upwards, the more anterior rest (the indirect retainer) resists downwards
movement and this increases the effectiveness of the direct retainer
o Class I and
Kennedy Classification

Bilateral edentulous areas located posterior to the remaining natural teeth

Unilateral edentulous area located posterior to remaining natural teeth

Unilateral edentulous area with natural teeth remaining both anterior and
posterior to it

A single, but bilateral (crossing the midline) edentulous area located anterior to
the remaining teeth

Pic of white plaque that cant be rubbed offleukoplakia


White lesion that cant be scraped off. May show benign hyperkeratosis, epithelial dysplasia, or invasive
carcinoma
Premalignant lesion. Tobacco, alcohol
H1 histamine and H2 (gastric)
H1: 1st generation sedative/hypnotic
Antihistamines
H1
Chlorphenira
Useful for treating
Antihista
mine
dermatologic
mines
manifestations of an
allergic response
Promethazine
Preoperative
`
medication for
sedation, antiemetic
properties,
anticholinergic

40

Diphenhydra
mine
H2
antihista
mines

Cimetidine

effects
For controlling the
symptoms of
parkinsonism
Used to reduce
gastric acid
secretion.
Now available OTC
for heartburn

You must know this Pg.451


- Dephendarmine, pyralamine, chlorpheramine, promethazine
- Hydroxzine used for pediatric anesth.
- H1: 2nd gen do not cross BBB.. non sedative
- Loaratide, Desloratide, Cetrizine, Actrivastine
- Fexotanadine (can not be taken w/Cimetidine)
H2 blockers
-Cimetidine, ranitidine, fmootidene, nizatidine tx: Gerd, ulcer, stop parietal secretion of H+
Cheek biting (dentures)
horizontal overlap
post teeth too edge to edge
If patient cant make F and V sounds, whats the problem? Too far superior and too anterior
Speech Sounds
Fricative/Labiodental Sounds

F, V, and ph sounds

Maxillary incisors contact the wet/dry lip line of mandibular lip

Determine the position of the incisal edges of the maxillary

If incisal edges of the max incisors contact the lower lip 1 mm or


more labial to the wet dry line when making f and v sounds, then the
max teeth may be set too far labially.
o Treat by resetting anterior teeth in a more lingual position.
The incisal edge of max incisors should contact the wet/dry
junction just lingual to it during production of f,v, sounds
Sibilant/Linguoalveolar Sounds

S, z, sh, ch, and j

Tip of tongue in most anterior part of palate or lingual surface of


teeth

Help determine vertical overlap of anterior teeth

if there is a whistle on the s sound, then air is passing unimpeded


between the dorsal surface of the tongue and the anterior palate
o treat by increasing palatal resin, convex contours lingually to
the max central incisors to impede the air stream passing
between the tongue and palate, create rugae if you have to

if there s a lisp on s sound- air stream is passing between the


tongue and interior palate is excessively impeded, usually by rugae
or excessive resin contour
o solve by decreasing VDO until premolars no longer contact
during speech
Linguodental Sounds

This, that, those

Tip of tongue protrude slightly between maxillary and mandibular


anterior teeth

Help determine the labiolingual position of anterior teeth


B,P, and M sounds

Made by lip contact

Insufficient lip support by the teeth or labial flange can affect these
sounds

Lot of questions on irreversible pulpitis/aap


o AAP: symptomatic, no radiolucency, percussion positive, pulp, tooth vital or nonvital EPT/pulp test most
imp. Way to confirm if vital pulp, simple occlusion adjustment will be enough tx if nonvital and untreated
will lead to acute apical abscess

41

o
o
o
o

Acute apical abscess: purulent exudates around apex, symptomatic, PDL maybe normal looking or slightly
thickened in xray, normal or slight thickened lamina dura, SWELLING rapid onset of swelling, mod. To severe
pain, pain w/percussion and palpation, slight inc. in tooth mobility
Chronic periradicular periodontitis: asymptomatic, radiographic visible, endotoxins cascading into pulp cause
extensive demineralization of cancellous and cortical bone, slight tenderness to percussion/palpation
Chronic periradicular periodontitis/phoenix abscess: similar presentation as acute apical abscess BUT
radiographic evidence/ periapical Radiolucency histo: liquefaction necrosis w/PMN, viable macrophages and
occasional lymphocytes and plasma cells,
Suppurative periradicular periodontitis/chronic periradicular abscess: draining sinus tract w/o discomfort,
mimic perio pocket, non-vital pulp, bone loss xray

Operative gypsum (setting time/working time)


The setting expansion of any gypsum product is a function of calcium sulfate dehydrate crystal growth. Some
is the result of thermal expansion
Gypsum classifications:
o Type I: plaster, impression plaster
o Type II: model plaster
o Type III: dental stone
o Type IV: dental stone, high strength die stone
o Type V high strength.
A thinner mix of gypsum base product decreases the degree of exothermia, decreasing setting expansion.
Increasing the water: powder ratio increases setting time and decreases strength.
Potassium sulfate and sodium chloride accelerates setting of gypsum, whereas sodium citrate and borax retard
setting
The poured cast should be allowed to set for 45 to 60 minutes before separating it from the impression
Investments expand during setting when heated.
o Gypsum bonded investments used for casting alloys containing 65% -75% gold at temps near 1100C.
they have a gypsum binder
o Phosphate bonded investments are used for casting metal ceramic crowns due to their capability to
withstand high temps (1,100 C). they have a metallic oxide and phosphate binder.
o Silica bonded investments are used for casting base metal alloys for frameworks for dental prostheses,
they have a silica gel binder.
Pat had two surface caries, where you smooth surface caries? Below the contact
Pat had mesial/distal lesion, would you compromise all the tooth structure? No.. MOD.. but if its a primary 1st Mand.
Molar.. pre-fab crown You must know this Pg.452
Curing light? LED? 400-499 wavelength minimum of 300
Selecting shade for pt, what do you consider? Value, chroma, hue
o 1st. Hue, 2nd: Chroma, 3rd Value
o chroma: the saturation or strength of a color
o value: the lightness or darkness in a color
o hue: color tone
o value is the single most important factor in shade selection. Intensity is included in the term value, the hue
should be selected first when picking a shade
Cantilever abutment? Worst prognosis
Canine as abutment for cantilever
After perio surgery, most important thing for success? No retained plaque keep plaque free envt.
After perio surgery, how does patient clean interproximal surfaces? Repeat question
In crevicular fluid, what cell do you find. 92% PMNs, 4% B-cells, 3% T-cells, 1% phagocytes
o initial lesion (gingivitis)PMN infiltration in junctional and sulcular epithelia
o early lesion (gingivitis)- lymphocytes predominate, rete pegs and atrophic areas in junctional and sulcular
epithelia
o established- plasma cells predominate, like stage 2 but more advanced.
After removing plaque (2days), what bacteria do you find? Fusobacterium nucleatum, Prevotella intermedia, and
Capnocytophaga species by a week time "tertiary colonizers", and includePorphyromonas gingivalis, Campylobacter
rectus, Eikenella corrodens, Actinobacillus actinomycetemcomitans, and the oral spirochetes (Treponema species)
initial species: S. sanguis, A. viscous, S. Mutans

42

C-factor
o The ratio of bonded to unbonded surface areas of a composite restoration.
Polymerization shrinkage in a composite creates stress that can damage surrounding enamel walls of the
cavity preparation. The amount of stress depends on the C-factor of the composite restoration. A high C-factor
indicates the cavity is more likely to be damaged. Incremental curing reduces the C-factor, and therefore
reduces the residual stress of the resulting composite restoration.
Which tissue is least radiosensitive? Neurons, skeletal muscle. Cells that are mitotically active are the most
radiosensitive (basal cells of the oral mucosa) Skeletal muscles are least radiosensitive
o

Cavulinic acid/augmentin incr. action of penicillin b/c calvunic acid is a beta-lacatamase inhibitor tx: H. influenza,
N. gonnorreha, E. coli, P. numococci
By combining clavulanic acid with a penicillin, the beta-lactamase enzyme is permanently inhibited by the
acid, and the antibacterial activity of thepenicillin is maintained
One popular preparation is augmentin which contains amoxicillin and clavulanate potassium. Augmentin is
used orally as pill or liquid form.
Sulbactam is another beta lactamase inhibitor
2nd 1

molar distal shoe


Missing primary first molar? Band and loop
Missing primary second molar? Distal shoe
Premature loss of primary mandibular canines? A Lingual holding arch will prevent the posterior teeth from
tipping mesially
Bilateral loss of max
primary teeth? Nance appliance
HMO/PPO
o HMO health maintenance organization is a type of managed care organization that provides a form of health
care coverage in the United States in which doctors and other providers have a contract with.
o PPO preferred provider organization is a managed care organization; doctors accept reduced fees in
exchange for referrals.
o Whats the most common reason patient needs to repair anterior composite? The majority of North American
dental schools reported marginal defects (84 percent) and You must know this Pg.453
o marginal discoloration (73 percent) as the most common indications for repairing RBC(resin based composite)
restorations
o Mjr and Toffenetti3 reported that secondary caries is the most common reason for replacing restorations in
general dental practice.
How do you treat a fearful patient?... ID: Fear
How long do you splint?
o Avulsed: 7-10 days (closed apex). 3-4 weeks (open apex)
o Luxatation: 14 days if alveolar process also damaged 4-6 weeks
o Subluxation: 7 -10 days if mobile
o Midroot/apical: 2-3 months
o Alveolar process: 4-6 weeks (rigid splint)
classification: know how many weeks/months for calcification
Primary teeth calicify: 14-18 mos in utero (A, D, B, C, E respectively)
Secondary teeth calcify: 6=Birth, Max central/Mand Central & lateral = 3-4 months, max lat- 10-12months,
Canines =4-5months,. Mandibular teeth are .25 yrs after Max. 1st pm = 1.5yrs, 2nd pm=2 yrs, 2nd Molar
= 2.5 yrs
how do you treat dry mouth?... sialogogues ex. Pilocarpine (direct acting cholinergic agonist)
Drugs for depression/seizure
Know what they work on

TCAs

Imipramine
Amitriptyline

Antidepressants
Block reuptake of NE
Amine reuptake
and 5ht
blockers
Block the peripheral
adrenergic and
muscarinic receptors
postural
hypotension,
Mydriasis, (Mydriasis

Anticholinergic or
atropine like side effects

43

makes it a
contraindication for
glaucoma)
MAOIs

SSRIs

Antimani
cs

benzodia
zepines

Tranylcypromi
ne
Phenylene
Fluoxetine
Trazodone
Lithium

Diazepam
(valium)

Midazolam

Barbitura
tes

Chlordiazepox
ide
Clonazepam
Thiopental

Blockade of amine
reuptake or alteration
of receptor numbers

Anticholinergic or
atropine side effects

Block serotonin
reuptake

Anticholinergic or
atropine side effects

Primary use is in the


treatment of the
manic phase of
depressive psychosis

Less addictive
potential than
barbiturates
Less profound CNS
depression than
barbs
Larger therapeutic
index
Less respiratory
depression
Water insoluble,
needs propylene
glycol to dissolve it
Short acting
compared to valium
because it doesnt
have active
metabolites like
diazepam.
The most lipid
soluble
benzodiazepine, so
rapid onset, short
duration

Barbs are not


analgesics
Barbs often induce
excess salivation and
bronchial secretions.
Have anticonvulsant
properties
Depress all levels of
the CNS
No significant
analgesic effects
Used for anesthesia,
anticonvulsant, and
anxiety
Can produce
unconsciousness in

xerostomia

Sedatives
Modulate the activity
of the inhibitory
neurotransmitter,
GABA (increase
GABA)

Form active metabolites


Injection of diazepam
can cause irritation such
as thrombophlebitis due
to the solvent the BDZ is
dissolved in
Flumazenil reverse the
effects of
benzodiazepines

Water
soluble (so
doesnt
need to be
dissolved in
propylene
glycol like
valium)

Thiopentals action is
terminated by
redistribution of the
drug out of the brain
it enters the brain
rapidly and exits
rapidly, thus quick
onset and short
duration of action
Inhibit depolarization
of neurons by binding
GABA receptors
Enhances
transmission of
chloride ions

Metabolized
by the liver
Excreted in
the urine

Carbamazepine is used
to treat trigeminal
neuralgia.
Carbamazepine is an
antiepileptic med used
to treat grand mal and
psychomotor seizures.

Barbiturate overdose
kills you because of
respiratory depression
Barbiturates are
contraindicated in a
patient with intermittent
porphyria- barbs
enhance porphyrin
synthesis and thus will
aggravate the disease
Sudden withdrawal from
a high dose can be fatal
Contraindicated in
pregnant patients and
patients with respiratory
diseases
Drug interactions with
CNS depressants,
alcohol, and opioid

44

under 30 seconds
(very lipid soluble, so
rapid onset of action)

analgesics enhances
CNS depression

After ortho tx, rotated tooth, what causes this supracrestal periodontal fibers
Definition of efficacy
o The number of receptors that must be activated to yield a maximal response.
o A drug with high efficacy needs to stimulate only a small percentage of receptors, while a drug with
lesser efficacy has to activate a larger proportion of receptors
The ability of a drug-receptot complex to produce a functional response.
Water on amalgam
o Is moisture is incorporated into an alloy that contains zinc, the water reacts with the zinc to produce hydrogen
gas, which causes severe expansion of the amalgam.
Overtrituration decreased expansion
Root caries/xerostomia ?
Max tuberosity and retromolar area touching. What do you do?... reduce tuberosity
EDTA chelating/green stains???
Know the signs of trauma
o Clinical signs include: increased tooth mobility, thermal sensitivity, attrition of enamel, recession of the facial
gingival tissue.
Difference between bur 245 and 330
o These two burs are very similar to one another. The 245 is 3.0mm long.
o 330 is shorter than 245
Preparing a veneer, in middle 3rd, how much do you reduce .5 mm You must know this Pg.454
Implants how far should they be apart 47deg. C or 117deg. F (critical temp.).. 10mm vert and 6 mm horiz.
o 3mm apart from each other
o

1mm from root of implant to tooth

1mm from facial and lingual

5 mm from mental

2mm from Inf. Alv

1mm from PDL

1mm Nasal vestibule

Too much Acetaminophen liver dz


Symptoms of down syndrome
Down syndrome has underdevelopment or hypoplasia of the mandible, its associated with class III
malocclusion. Patients often have open bites, tooth eruption is delayed. The tongue is often fissured, and

45

macroglossia is usually relative to the small oral cavity. Dry cracked lips often result from a protruding tongue
and frequent mouth breathing
Patients with Down syndrome and heart defects often require subacute bacterial endocarditis prophylazis

Patient was dissatisfied with shade of crown. Dentist likes it. What do you do?.... surface characterization..
Remineralization of a tooth. How does it feel compared to a tooth that is normal?... feels rough but hard like normal
Furcation involvement of a mandibular molar.
o Hemisection.
o

Treat like two premolars

3 wall, narrow vs a 2-wall shallow crater greater success rate for 3-wall defect
Pic of residual cyst

RL with RO borders found in edentulous areas where a tooth with a radicular cyst was extracted but not
curettached.

2 xrays, identify max sinus, orbit, zygomatic process

Above pic identifies the inferior border of themaxillary sinus

The zygomatic process of the maxillar protrudes laterally from the maxillary wall. On pas it appears as a u
shaped radiopaque line with its open end directed superiorly

46

1. articular tubercle, temporal bone


2. Zygoma
3. Zygomatic process of the maxilla
4. Pterygomaxillary fissure
5. Floor of orbit
6. Anterior aspect of inferior concha
7. Nasal septum
8. Anterior nasal spine
9. Floor of max sinus
10. Maxillary left third molar (developing)
11. Ear lobe
12. Cervical vertebral body
Pic on gingival inflammation
Pic of patient wearing denture. Candidiasis

47

Papillary hyperplasia (pictured to the right) is caused by local irritation,


poor hygiene, ill fitting dentures, and leaving dentures on in the mouth
all day and night. Usually occurs on the hard palate beneath a denture
base

Denture stomatitis (chronic atrophic candidiasis, bottom lefr) not a true


infection

Epulis fissuratum: a tumorlike hyperplasia of fibrous CT that develops in


association with the flange of an illfiting complete or partial denture (seen
below)

Wax up for investing, casting,


A dental investment is a refractory material that is used to surround the wax patter during the procedure of
fabricating the metallic permanent restoration. It forms the mold into which the alloy is cast after the was has
been eliminated
An investment material to be used for a casting mold should expand on setting and heating to compensate for
the shrinkage of molten metal as it solidifies.
o Gypsum bonded investments, are used when casting conventional gold alloys containing 65-75% gold
at temps near 1,100 degrees celsius
o Phosphate bonded investment:

Type I is used when casting base metal alloys for metal ceramic crowns

Type II is used for removable partial denture frameworks. Are capable of withstanding
temperatures higher than 1,1000 degrees celsius
Porosity in pulpal floor. What will it effect?... coping base will not sit properly
What drug to treat herpes? An antiviral agent penciclovir (denavir) is active against the herpes virus. It is a cream
indicated for the treatment of recurrent herpes labialis (cold sores) in adults. It inhibits viral action by selectively
inhibiting herpes viral DNA synthesis and therefore resulting in the inhibition of viral replication. Other agents indicated
for use in treating the condition of herpes labialis are: acyclovir tablets, cream, docosanol cream (abreva), lysine
tablets, and valacyclovir (valtrex)
Sealants
Pt had an implant 2 stage after there was bone loss and mobility. bone graft.
Histodifferentiation stages (development) know all of this.
Problem of the bell stage (Histodifferentiation, Morphodifferentiation)

48

o
o
o
o
Analgesic that

Leads to Macrodontia and Microdontia


Peg lateral incisors
Dentinogenesis imperfecta
Amelogenesis imperfecta
lasts 8 hours. Naproxen (over the counter NSAID Alive)

Problem with a drug and a dental device, who do you report it to: FDA You must know this Pg.455
Black female with periapical dysplasia
Periapical Cemental Dysplasia

Apex of vital teeth


Black women most
Middle age (40s)

Solid RO mass
surrounded by a RL
border
Depending on stage it
can appear RL or totally
RO
Opacities are bone

Anterior mandible

Periapical Cemento-Osseous Dysplasia*


Side effects of benzodiazepines. Flumazenil for benzodiazepine overdose
dependence
You must know this Pg.457 Clinically there is a green discoloration at the margin of a PFM crown? What causes this?
Copper, zinc, palladium, cobalt? Contamination of metal by copper traces causes porcelain crown to have green
discoloration when baked onyo a high fusing alloy
Definition of hypertolerism
Abnormally increased distance between the eyes
Feature of crouzons,, nevoid basal cell carcinoma,
Patient visited doctor and had an HbAC1 of 12. Patient also had uncontrolled periodontitis. What is your course of
treatment? Request medical consultation from doctor, Scaling & Root planning, premedicate and scaling and root
planning?
A form of hemoglobin used primarily to identify the average glucose concentration over prolonged periods of
time. It is formed in a non-enzymatic pathway by hemoglobins normal exposure to high plasma levels of
glucose.
The American Diabetes Association recommends that the HbA1C be below 7.0 for most patients.
A high value represents poor glucose control.
Tons of questions on Cohort studies and longitudinal studies (Public Health Section of Mosbys)
Cohort studies-prospective and retrospective, studies follow a general population over time for prevalence of some dz
or the other way around for latter
o Longitudinal ecologic studies use ongoing surveillance or frequent cross-sectional studies to measure trends in
disease rates over many years in a defined population. By comparing the trends in disease rates with other
changes in the society (e.g., wars, immigration, or the introduction of a vaccine or antibiotics), epidemiologists
attempt to determine the impact of these changes on the disease rates.
o Questions on chroma-intensity of the hue, value-relative lightness and darkness of a color, and hue- dominant
color of an object (Red, blue, green)
First order and second order bends (ortho)
First order bends are in horizontal planes
Second order bends are in the vertical plane
Third order bends are in the buccal-lingual direction
Best of Luck
Afuray!

This is my no sleep PTSD regurgitation, some of them are whole questions, most are just the concept with an answer
choice or 2. The answer is probably one of the 2 that I narrowed it down to. Also my spelling is embarrassing enough to

49

think English isnt my first language. Good luck, I make no promises about right answers and expect unmarked bills in
a discrete envelope.
Jan 2013
Which type of sutures wick? Silk, gut, e-ptef, nylon
The primary purpose of splinting? Pt comfort, prevent mobility, etc
What teeth classify dental age 11?
Max k9 and 1st pm
All second premolars
Whats the most common cause of kid Heart attacks
Resp depression
What number do u not want your curing light to go below? 400-499
Kid ingested 20mg NaF what happens? Coma, nausea, kidney failure,
Most frequent erupting perm tooth in x bite
max lateral incisor
An anterior crossbite may indicate skeletal growth problem and a developing class III maloclusion
Posterior crossbite often correlates to a narrow maxilla
o Unilateral posterior crossbite often determines a lateral shift of the mandibular position which can lead
to skeletal asymmetry (from unilateral maxillary constriction)
Most freq impacted tooth
Mandibular third molars > maxillary permanent canines
Dentist waives the difference bt cost and insurance payment, without telling insurance whats it called? Unbundling,
price fixing, over charging,

Accepting your insurance as payment in full and disregarding your coinsurance, copayment or
deductible results in overbilling the insurance company. Its against the law in many states and is viewed
by the dental profession as unethical. Its also a violation of the dentists contract with Delta Dental.

Dentists are not reducing their fee when they do not collect patient payments. Instead, these dentists are
charging inflated fees to the insurance carrier to make up for the money they lose from waiving the
coinsurance/copayment amounts.

Enrollees who knowingly agree to the dentists scheme are participating in this deception.

What is the conditioned stimulus? Dental chair


Classical condition: a neutral stimulis is paired with an unconditioned stimulus
o Ex. A dentist gives a painful injection and the patient experiences anxiety and upset. The stimulus will
become the dentist
According to research primarily anxiety comes from? Family/friends, past experience, media, perceptions
Systematic desensitization, which isnt true? Identify fear, pair with hierarchy, teaching relaxing stageties,
Systematic desensitization: exposing a patient to items from a collaboratively constructed hierarchy of slowly
increasing anxiety-provoking stitmuli (related to target fear) while using relaxation skills
Backbone of informed consent? Autonomy
Most likely place for carries in primary teeth? D of 2m, d of 1m, m of 1m, m of k9 dont know (according to mosbys is
the distal surface of primary K9)
How do x ray produce most bio damage? Ionization of dna direct, indirect, release of free radicals direct, indirect
direct effect: direct alteration of biologic molecules (DNA, lipids, proteins) by ionizing radiation
o 1/3 of the biological effects of x-ray exposures result from direct effects
Indirect effect: radiation effects mediated through water
o 2/3 of radiation induced biological damage results from indirect effects
changes in biologic molecules (nucleic acids)
o damage to the DNA molecule is the primary mechanis for radiation induced cell death, mutation, and
carcinogenesis
What would you give Iv to stop life threating ventricular arythmia? Lidocaine, propranalol, 2 others
when lidocaine is used intravenously to treat ventricular arrhythmias, it acts on the fibrillating ventricles to
decrease the cardiac excitability and spare the atria. It can reverse a life threatening situation
acebutolol (sectral) is a beta 1 selective adrenergic receptor blocker used to treat hypertension and to control
ventricular arrhythmias.
o Beta 1 agonists will increase cardiac output
o Beta blockers are only used for patients who have mildly symptomatic ventricular arrhythmias
o With all spelective beta blockers, selectivity for the beta 1 receptor is lost at high doses.

50

Propafenone is an antiarrhythmic agent used to treat both ventricular arrhythmias and supraventricular
tachycardias
Which dont you see a dev delay? Edwards syndrome, treacher Collins, cri-du-cat, hurler
Treacher Collins: AD, downwards turned ees, hypoplastic zygoma, mandibular coronoid hypoplasia retruded
chin
Edwards syndrome: kidney malformation, structural heart defects, developmental delays, fetuses often
dont make it to birth
Cri du chat: feeding problems severe cognitive, speech, and motor delays, wide eyes, small head and jaw
Give a ianb and you get hematoma, what do u do first? Pressure, heat,
Outcome of intrapulpal injection? Anesthesia via back pressure, anestehesia after 30 sec
What feature of max incisors aids in plaque retention? Palatoginival groove, cingulum,
On central cingulum, on max latera,s its gingival palatal groove

Identify inverted y on pa

\
the anterior wall of the max sinus makes the inverted y landmark (3, 4, in the picture)

Inverted Y Radiographic landmark made up of the lateral wall of the nasal Fossa
and the anterior-medial wall of the maxillary sinus often observed near the caninepremolar region

Which part of x ray machine does therminomic emission come from? Anode, filament , cup,
Thermionic emission is the heat induced flow of charge carriers (electrons) from a surface or over a potential
energy barrier.
The x ray tube consists of a lead glass housing, a negative cathode, and a positive anode
o Electrons are produced in the cathode and accelerated towards the anode.
o The anode converts the electroms into x-rays
o The cathode consists of a tungsten wire filament ina cup shaped holder of molybdenum

Tungsten filament: a coiled wire which produces electrons when heated

Molybdenum cup: focuses the electrons into a narrow beam and directs the beam across the
tube towards the tungsten target of the anode
Main source of radiation in US? Inhaled radon (53%), cosmic, terrestraial,
Natural radiation (background radiation) is the largest contributor (83%) to the radiation exposure of people in
the US. Background radiation results from external and internal sources
o External: radiation originating in the environment (terrestrial or cosmic). These sources contribute
about 16% of the radiation exposure to the population
o Internal: inhaled radon (56%) and ingested radionuclides (11%)
Major source of overlap in bw in kids? Incorrect film placement, incorrect horz angle
Incorrect horizontal angulation causes overlapping (teeth are superimposed on each other)
Which drug can u use with mystenia gravis? Penicillin, erythro, clarithromycin, imipimen I guess imipramine (TCA)
because it has anticholinergic effects
Neostigmine: reversible anticholinesterase or cholinergic agonist, extended duration of action, used to reverse
curare type drugs and treat myasthenia gravis
Physostigmine: reversible anticholinesterase or cholinergic agonist; extended duration of action, used to treat
myasthenia gravis
A cholinesterase inhibitor will be used to treat myasthenia gravis because they act by blocking
acetylcholinesterase that degrades acetylcholine in the brain resulting in more acetylcholine in the synaptic

51

cleft and enhanced cholinergic transmission. These drugs are indirect agonists at muscarinic and nicotinic
sites.
What will cause fatal reaction with narcotic? MAOI, TCA
The most common side effect of narcotics is nausea, the more serious side effect of narcotic analgesics is
repiratory depression (leads to death)
Alcohol is synergisti with narcotics, barbiturates, and phenothiazines
``
Meperidine (opioid analgesic) is most abused by dental professionals, its contraindicated for patients on
MAOis
Difference bt parent drug and conjugated (polar water soluble) form: less hydrophilic, more hydrophobic, more ion
form in plasma, increased action
Phase I reaction: occurs in liver microsomal enzyme system. First thie active parent drug is converted to the
inactive metabolite. Second, an active compound which is then converted to an inactive compound. Third, an
inactive parent drug may be transformed to an active compound.
Phase II reaction: conjugation reactions involve coupling the drug with an acid present in cells. When coupled
to glucuronic acid, the process is know as glucoronide conjugation with resulting metabolite referred to as the
glucuronide. Conjugation occurs in the liver, kidneys, and to a lesser extent in other tissues
Conjugation of drugs results in polar, water soluble compounds that are rapidly excerted in urine. Thus the
parent drug is ffectively rendered inactive and transported out of the body by this process
T/F gingivectomy incision with incision line coronal to base of pocket
Gingivectomy is indicated for elimination of suprabony pockets, elimination of gingival enlargments, and
elimination of suprabony periodontal abscess
Do not do gingivectomy if base of pocket is located at the MGJ or apical to the alveolar crest (must consider
pocket depths)
What is the min age for speech congintion(something like that) 3, 5, 8, 11
1 wk post extraction pt had 4 mm antro oral communication: watch 3-4 wks, buccal sliding flap, spit thinkness rotated
palatal flap, gold foil lining.
if sinus communication should occur prescribe afrin(nasal decongestant), amoxiicillin, and Actifed (a systemic
decongestant)
if the opening is of moderate size (2-6 mm) a figure 8 suture should be placed over the tooth socket.
If the opening is large (7mm or more, the opening should be closed with a flap procedure)]if the entire tooth or
a large fragment of one is displaced into the sinus, it should be retrieved through a Caldwell-luc approach asap
If a small communication is made during extraction, the best treatment is to leave it alone and allow the blood
clot to form
At what point do you see marginal leakage with class 2 composite w/o rubber dam? 4-6 wks, 4-6 mo, 1 year+, same
rate as w/o rubber dam? This is my guess
The effectiveness of an ultrasonic is dependent upon? Vibration, sharpness of instrument, sharp cutting surface or firm
pressure against the tooth.
Ultrasonic instrumentation is accomplished with a light touch and light pressure, keeping the tip parallel to the
tooth surface and constantly in motion. The working tip must contact all aspects of the root surface to remove
plaque and toxins throroghly.
Vibraations at the tip of the ultrasonic range from 20,000 to 45, 000 cycles per second. Sonic instruments
vibrate at a range of 2000-6500 cps at the tip, which provides less power for calculus removal than ultrasonic
units
You have a void in the middle of pulpal floor before everything (casting, investing etc) crown will? Dont know
Fit die but not tooth, tooth but not die, neither, both.
Main feature of major connector? Rigity and retention, support and stability,
Major connector must be rigid so that stresses applied to any one portion of the denture may be effectively
distributed over the entire supporting area. It connects other compoenents of the prosthesis and provides
cross arch stabilization
A minor connector is the connecting link between the major connector and other units of the prosthesis like
clasps, indirect retainers and occlusal rests
Major and minor connectors must be rigid in order for the functional stresses that are applied to the partial
dentures to be distributed evenly throughout the mouth
Indirect retatiners (Rests): prevent downwards movement and allow for retention. An indirect retainer should
be placed as far from the distal extension base as possible in a prepared rest seat on a tooth capable of
supporting its function. Indirect retainers assist the direct retainers in preventing displacement of a distal
extension base by functioning through lever actions on the opposite side of the fulcrum
Direct retainers (clasps): purpose is retention.
Fetal AS what do you see? Midface deficiency
Dentist control crown factor? Parallelism
When dont u want to use narcotics? Sever head trauma,
MOA acyclovir: something with phosphorylation (answer)
Inhibits viral DNA polymerase after undergoing phosporylation
Tooth with most likely 3 canal? Max 1pm, mand 1 pm, max 2pm, mand 2 pm

52

Max 1 pm: 3 canals 6%


Mand 1 pm: 3 canals: 0%
Max 2 pm: 3 canals 1%
Mand 2 pm: 3 canals 0%
Infection from mand 3 m, erodes lingual coretex, where does it go? Subling, pterygomax, submental or submand space
Sublingual space from mandibular molars and premolars
Pterygomaxillary space- mandibular molars and premplars
Submental space mandibular incisors
Submandibular space- mandibular molars and premolars
Masseteric space mandibular third molars
If a third molar disappears during extraction, its most likely in the submandibular space
Direction of fibers for implant CT? parallel, perpendicular
Implants have a biologic width of 3-4 mm
Connective tissue fibers are present as circular fibers oriented parallel to the implant or abutment surface, but
no connective fibers insert into titanium
After brushing how long until forming mature plaque reappears? 1-2hrs, 6-8, 24-32 hrs
Day 1-2 young plaque consists primarily of cocci (strep mutans and sanguis)
Day (2-4) cocci still dominate, but there are increasing numbers of filamentous forms and slender rods
Days 4-7 filaments increase in umebers, and a more mixed flora begins to appear with rods, filamentous forms,
and and fusobacteria.
Days 7-14: vibrios and spriochetes appear and the number of white blood cells increase. More gram negative
and anaerobic organisms appear. The signs of inflammation are beginning to be observable in the gingival
Days 14-21: vibrios and spirochetes are prevalent in older plaque, along with filamentous forms. Ginvitis is
evident clinically
As plaque ages, the number of cocci decreases, and the number of rods, fusiforms, filaments and spirochetes
increases. Anaerobes and gram negative organisms increase as well
Early cononiser of early plaque? Strep sanguinis, strep salivarious
Hazard communication standard: OSHA
Most common site site for recurrent hepies: hard palate, soft palate,
Secondary or recurrent herpes simplex appears on the lips, hard palate and gingiva
Thermionic emission: The heating of a metal wire to a temperature at which electrons in the wire gain enough energy
to escape to the space surrounding the wire already explained
Where is lichen planus? Buccal mucosa
Lacelike white striae (wickhams striae is bilateral and symmetrical). Appears more in women then men.
Mainly affects buccal mucosa. May also be seen on the tongue, lips, hard palate and gingiva. The striae is
usually asymptomatic. Intraoral lesions respond to topical steroid therapy.
Gorlin syndrome: cysts of the jaw, osteomas, (I think okc)
Gardners has multiple osteomas and intestinal polyps, odontomas (from decks), and supernumerary teeth
Gorlins has okcs, palmer pitting, bigid rib, multiple basal cell carcinoma
52 yo woman ulcers for 6 mo on max and mand gingival: phemphgus, apthous stomatitis
I would guess pemphigus. Usually in middle aged and older patients. Multiple oral blisters and ulcers usually
precede skin lesions
Gingival hyperplasia: vermapril (CCB)
Calcium channel blockers have been associated with gingival hyperplasia
o Calcium channel blockers end in dipine
Ex of calcium channel blockers: verapamil, nifedipine, diltiazem
What med for anti convulsant also approved for neuogenic pain: Carbamazepine treats trigeminal neuralgia
Max Ci sloped and notched from what? Cong syphilis
picture shown is hutchinsons incisors of congenital syphilis.
Hutchinsons triad includes
o Hutchinsons teeth
o Ocular interstitial keratitis
o Eighth nerve deafness

What stages are syphilis infectious? 1, 2,3, any combo of


Primary: first symptom is a nonpainful chancre that appears 2-6 weeks after exposure, its generally found on
the part of the body exposed to the partners ulcer (like the penis, vagina, lip, tongue) the chancre disappears
within a few weeks whether or not person is treated
Secondary : highly infectious stage; occurs 6 weeks after non-treatment of primary syphilis. Widely
disseminated spirochetes cause mucous membranes to exhibit a reddish brown maculopapular cutaneous rash
and ulcers are covered with a mucoid exudate. Condyloma lata (elevated broad based plaques) are also seen

53

on skin and mucosal surfaces. If left untreated, these symptoms will resolve on their own, but the infectious
microbe remains behind. It is at this point syphilis passes into the latent phase.
Tertiary: occurs in infected persons many years after non-treatment of secondary syphilis. The guma (a focal,
nodular mass) typifies this stage. It most commonly occurs on the palate and tongue. The bacteria damage
the heart, eyes, brain, nervous system, bones, joints or almost any other part of the body. Headache, stiff
neck, and fever are symtpoms of neurosyphilis
What do u see in a opioid abuser? Xerostomia, mydriasis, loose bowels,
Symptom of subacute Hg poison? Diarrhea, tinnitis, hair loss, weight loss
Excess saliva
Mercury that is absorbed into the circulatory system may be deposited in any tissue. Higher than average
accumulationsoccur in the brain, liver and kidney.
Restorative concern for primary molars? Divergent roots, wide Occlusal table, shallow pits
The occlusal table is narrower on primary molars
Primary teeth have thinner enamel
The pulp chamber is larger in primary teeth
The pulp horns are closer to the surface of the tooth
The crown is shorter and has a greater constriction in the cervical region
What metal is most allergic? Nickel
Nickel and Beryllium in the base metal alloys are allergenic
Which way does the max ridge resporb? Up and back, back, forward, forward and down
What do you not do about internal resporption? watch it (ans)
Bone composition of woman with osteoporosis? Dec calcification of osteoid, mosaic bone,
What cells responsible for local destruction in period x? b cells, t cells, fungal, virus
What is the best indicator of perio stability for maintance pt with chronic perio? Attachment level, probing, bop, hygine
What kind of sterilization for carbon steel carbide burs doesnt corrode? Dry heat only, ethylene, dry and gas,
autoclave
B/w pts how do u clean surface? Preclean then disincetant and leave for 10 mins, spray and wipe down, preset up 10
mins before pt and clean,
Whats most moisture tolerant imp material? Hydrocolloid, polyether, pvs, polysulfide
Sequence of steps to repair porcelain with composite: some version of silane, sandblast, etch and bond
You have a cast crown that seats on 30 that mand deviates to left when biting, what surface was premature? Buccal
inclines, lingual incline, mesial,
What systemic condition cant pt handle epi? Addison, hyperthyroid, adreanal insuff, you want to be carful with
administering epinephrine to patints that have hyperthyroid because it will elevate their blood pressure and cardiac
strains.
True about fluride cavit prevention/mineralization? Can remineralize without apatite crystals, prevent demin during
acid challenge,
Hyperventilation syndrome has dissiness and what? Dissiness and confusion, tacky cardia and tackypenia, tackypenia
and bradycardia,
Weakness, confusion, fainting, agitation, chest pain and shortness of breath
Best place to place implants? ant mand
Chronic caries, what you see clinically? Pigment, extensive undermined enamel, easy elicit pain,
Max ci with radiograph no pulp chamber, what from? Trauma, physiologic age,
Restore with amalgam class 2, where is the band prox? 1mm higher than MR
Whats most powerful and longest lasting? Pregnisone, cortisol, hydrocodisone, Dexamethasone
Dont get consent? Battery
Truth=veracity
Dentist with HEMA=contact dermatitis
The lack of secrtions from Benadryl from? Anticholinergic, antihistamine, anti adrenergic
Graft from different species=xenograft
Anug with hiv pt, what do u do? Debridement and antibacterial meds
Which osseos defect would GRT and bone be least effective? One wall, two wall, three narrow, 3 wide
Most neoplasms in what glands? Parotid, minor, subman, subling
Pt disoriented and hypoglycemic give what? Glucose, epi, steroids, insulin
Conscious fruit juice
Unconscious- 50% Dextrose IV
Doing dental work on type 1 diabetic when? 2 hr after meal and insulin, 2 hrs after meal no insulin, no meal or insulin,
2.5 yr yo with 12 cavities, do what? Nitrous and local, GA, nitrous only
2.6 Which med should not be used in anxiety control for a child: choices were N2O, Valium, Hydroxizine,
Midazolam (Answer choice = ??? All of them can be used, as far as I know. The child in the question

54

wasnt taking any medications; had no health problems. Had history of tonsillectomy and one other
ectomy surgery that Id never hear of.
Implant with internal component for what? Anti rotation
What is true about a threaded post for amalgam? 1 mm in axial direction of tooth, 1.5 mm axial, parallel to outer
contour, perpendicular to pulp chamber (stupid q)
Reason for functional cusp bevel? Structural durability, resitance,
Pfm crown with opaque area in incisal 1/3, why? Inadequate 2 nd plane reduction, over reduction of incisal, too much
body porcelain,
Pt syncope how to stop them? Head below heart, stand, supine, prone,
Pt swallows crown do what? Sit up or stand up, supine, prone, semi supine
Taking CR what position? Supine, semi supine, sitting up/standing,
Best biopsy for 2x3x2 white lesion? Incisonal, excisional, brush, asp
Biopsy to identify candida? Brush
Pt with general muscle weakness see what? Low mand plane angle ,
Most common cause bells palsy: hsv, idioscratic, parotid malig,
Generalized max denture soreness? Candida, vitamin def, allergy, gross Occlusal discrepancy
Excess monomer=shrinkage
How long does pain have to persisit to be chronic: 1-3 mo, 3-6 mo, 7-12 mo, 12+
Full thickness flap, what bone resporbs after? Thin interdental, thick interdental, thin radicular, thick radicular
Whats not characteristic of Mod Widman flap? Margins
Mosbys p257 This flap is not reflected beyond the mucogingival line. This flap designs allows for removal of the pocket
lining and exposure of the tooth roots and alveolar bone, but does not allow for apical repositioning of the flap.
What is the purpose of a protrusive measurement? Ant guide table, horizatal condylar angle, horizontal
What is the point of drying sulcus? Ease of cord placement, not diluting hemostatic agent, prevent bleeding
Narcotic analgesic with what is lethal? MAOI
Nonsedating antihist: Loratidine (Claritin)
Characteristics: long half-lives, do not readily cross blood-brain barrier, little or no sedation, higher risk of cardiac
arrhythmia(long QT effect) and drug-drug interaction with astemizole and terfenadine.
Antimicrobial therapy helps perio how??? Shrinkage, reattachment, regeneration, resection
Systemically administered NSAIDS inhibit the formation of prostaglandins (PGE2)
Subantmicrobial dose(doxycycline) inhibit MMP destruction of collagen.
Best thing to do for overdenture hygiene? Topical Fl drop 1xday, cast coping, amalgam core,

Maintenance and After Care (extremely important!)


o Strict instructions must be given
o Oral hygiene procedures

Brush denture after each meal

Soft tooth brush

At 45 degree angle, vibrating motion

One inch wide gauze

Wrap it around the abutment toothclean in a shoe-shine fashion

Floss

to be used under the bars


o Use of fluoride gel inside the denture--.4% stannous fluoride gel

This is indicated in irradiated patients (remember that one indiction for these dentures are for
cancer patients

Placed once a day in the sockets where denture overlaps tooth


T/F remove amalgam overhang as part of initial therapy
February 4th, 5th
Endo
Which one of these tests tells you nothing about the health of the pulp.
o Endo ice
o Ept
o Palpation
o Percussion
o The ept fails to provide info about the vascular supply to the pulp, which is the true determinant of
pulp vitality (decks)
o Electric pulp testing does not suggest the health or integrity of the pulp; it simply indicates that there
are vital sensory fibers present within the pulp (page 7 mosbys)
If doing an ept test which teeth should you use

55

o The teeth adjacent to the suspected tooth and the equivalent teetth in the same arch
o Same tooth in different arch and adjacent tooth
Know slob rule! I had several questions on his
o Be able to apply it,
o If the x ray cone is shifted mesially, the lingual palatal root/ lesion will shift to the mesial side
You performed an mod restoration on a patient a month ago. She comes back and reports that she has been
having incredible pain when she bites.
o Cracked tooth
o Know that cracked teeth have pain only on upon release of biting pressure. patient with cracked tooth
syndrome patients may experience sensitivity to mild stimuli like sweet or acidic food, and also cold.
You can use transillumination or tooth sloth in order t diagnose
In what situation is RCT contraindicated
o VRF
o this is a hopeless prognosis. Diagnose by visualizing the fracture with an exploratory surgical flap. In
a lot of cases there is an isolated probing defect at the site of the fracture. Will present
radiographically as a J shaped lesion.
Mandibular access opening is trapezoid shape
Edta is a chelator which removes inorganic material
Which is not a benefit of a steel file vs. a nickel-titanium file.
o More flexible
o Less chance for breaking
o Allows the file to be centerd in canal before separation.
o Niti files are more flexible, stainless steel shows more chance o breaking. Stainless steel file I more
prone to show signs of fatigue (from pals)
Why will you recapitulate with a smaller file in between files
o To clean the apical 1/3 of the tooth that wont be cleaned with just irrigation
How can you diagnose horizontal fracture?
o Angle the cone horizontally at multiple different angles
o Angle the cone at various vertical heights to capture the tooth at different vertical angles
o Since root fractures are generally oblique (facial to palatal) one pa radiograph may miss it, so the
radiographic examination should include an occlusal film, and 3 pas (one at 0 degrees, then one at +
and 15 degrees from the vertical axis of the tooth.
Had a patient with a horizontal apical root facture (it was in the apical third) the apex was closed, tooth had no
mobility. How do you treat
o Monitor for a year
o Rct
o Coronal fractures have a poor prognosis, midroot fractures have a guarded prognosis, and apical
fractures have the best prognosis
o Horizontal fracture is better than vertical, nondisplaced is better than a displaced fracture, and oblique
is better than transverse
o I think you just monitor this for a year a coronal root fracture, youd splint for 6-12 weeks, a mid root
facture youd stabilize for 3 weeks. An apical root fracture has the best prognosis they didnt t
specify a treatment
Know procedures for pulp capping
o Pulp capping procedure is most successful in accidental exposure of the pulp, and in the pulp of a
young child. Pulp capping is most successful if the exposure was accidental (trauma with a dental bur
as opposed to carious. The pulpal exposure should be only pinpoint to expect success. Repair is
accomplished by the formation of a dentin bridge at the site of exposure. Even a small carious
exposure should have root canal therapy for the best long-term prognosis. Young pulps are more
vascularized and therefore more amenable to repair.
o Indirect pulp capping involves removing infected dentin almost to the point of pulpal exposure. CaOH 2
is placed and then a resin modified glass ionomer cement is placed over that. Formation of a
secondary dentin should occur and then a final restoration is placed after removal of the intermediate
restoration and residual caries. The goal of indirect pulp capping is to have the tooth participate in its
own recovery. Indications for indirect pulp capping include deep carious lesions that encroach but are
not actually in the pulp, no history of chronic pain, no radiographic pathology, vital pulp, and normal
tooth mobility and color
o Direct pulp capping is indicated if there has been a small mechanical exposure, if it is an astmptomatic
vital pulp, and there is no coronal or periapical pathology. A hard tissue barrier (reparative dentin
bridge may be visualized as early as 6 weeks postoperative
Causes of endodontic failure
o Obturating material is overextended
o Lateral canals
o The main causes of endo fails is inadequate seal of the root canal system, poor access cavity,
inadequate debridement, missed canals, and procedural errors (perforation, ledging, loss of length)

56

Patient reports pain on chewing in maxillary posterior region


o Maxillary sinusitis
o The maxillary sinus is the most common location for sinusitis, and its associated with all of the ommon
signs and symptoms, but also results in tooth pain, usually in the molar region
Why is ZOE a good temporary restoration
o It provides a good seal
o Withstands compressive forces
o ITS NOT easy to reamove
o Most root canal sealers are some type of zinc oxide eugenol cement, and are capable of producing a
seal while being well tolerated by periapical tissues
Know this lots of questions on this:
Acute periradicular periodontitis: painful inflammation around the apex (localized inflammation of the PDL) can
be the result of pulpal disease extending into the periradicular tissue, canal instrumentation or overfill,
occlusal trauma like bruxism.
o Tooth can be vital or non vital

If tooth is vital, a simple occlusal adjustment will often relieve the pain

If the pulp is necrotic and remains untreated, additional symptoms may appear as the disease
progresses to the next stage- acute apical abscesses
(acute apical abscess): a painful, purulent exudate around the apex
o It is a result of the exacerbation of acute apical periodontitis from a necrotic pulp
o Radiographically may see a normal or slightly thickened lamina dura
o Symptoms:

Rapid onset of swelling, moderate to severe pain, pain with percussion and palpation, slight
increase in tooth mobility,

The acute apical abscess can be differentially diagnosed from the lateral periodontal abscesses
with pulp vitality and testing, and sometimes with periodontal probing
Chronic periradicular periodontitis: a long standing, asymptomatic, or mildly symptomatic lesion
o Usually accompanied by radiographically visible apical bone resorption
o Diagnosis is confirmed by:

The general absence of symptoms

The radiographic presence of a periradicular radiolucency

The confirmation of pulpal necrosis


Supurative periradicular periodontitis (chronic periradicular abscess)
o Associated with a draining sinus tract without discomfort
o The exudate can drain through the gingival sulcus, mimicking a perio lesion with a pocket
o Radiographic examination shows the presence of bone loss at the periradicular area
o Sinus tract resolve spontaneously with nonsurgical endo treatment
Chronic focal sclerosing osteomyelitis (condensing osteitis)
o Excessive bone mineralization around the apex of an asymptomatic vital tooth
o This radiopacity may be caused by low grade pulpal irritation
Mandibular molar access opening is- trapezoid shaped
In endo perio lesion, treat endo first.
Why would you put a dowel on an endodontically treated tooth

Operative
In a class II lesion, the caries are located apical to the contact point
When would you place a wedge
o After youve placed the rubber dam
o After youve placed your matrix band
o The purpose of the wedge is to compensate for the thickness of the matrix band. It will ensure a
positive contact relationship after the matrix is removed following the condensation and initial carving
of amalgam
Know that when correcting an indirect exposure, or correcting a direct pulpal exposure, the procedure for
placing calcium hydroxide. They asked a couple of times would you follow calcium hydroxide with a base.
Asked which base. Also asked for the thickness of that base.
o Calcium hydroxide is a liner. Its used for pulpal protection. Indications for use:

Only used when you have exposure or possible exposure

.5 mm thickness
o RMGI is recommended as a base to overlay any calcium hydroxide liner that has been placed. This
base provides additional strength to resist amalgam condensation pressure as well as protection of the
liner from dissolution during etchant application for bonded procedure. (place 2mm thickness)
Question on C curve
What material will you use to restore a class V lesion that extends to the root (geristor wasnt an option)=
RMGI
What surfaces will you bevel on a class V lesion = all surfaces with enamel

57

When removing amalgam restoration, what is the best way to keep free amalgam from entering into the
atmosphere
o Use rubber dam
o Use saliva ejector
o Use high speed suction
o I said high speed suction
Indications for gold onlay
o Large occlusal surface, provides bracing for root canal treated teeth, bridge retainers, partial retainers
Whats the best restoration to oppose a full ceramic crown? I put a gold onlay
Most likely to cause an allergic reaction- nickel
Know treatment planning phases
o Urgent phase
o Control phase
o Reevaluation phase
o Definitive phase
o Maintenance phase

Prosth
What helps titanium oxide in osseointegration- I put roughening the surface (sandblasting I think) in order to
increase surface area I dont remember the other options.
The purpose of a facebow is to relate the maxilla to the
o Terminal hinge position
o Condyles
o The purpose of the facebow is to orient the maxillary cast to the hinge axis on the articulator
If the occlusal plane is too low, what does that cause?
o Biting of tongue
o Increased pressure to the ridge
o Couldnt find this answer all I found is insufficient vertical dimension causes an aging appearance of
the lower third of the face due to thin lips, wrinkles, chin too near the nose, overlapping corners of the
mouth.. it also causes diminished occlusal forces, and angular cheilitis.
o Had a question about correcting condylar inclination from 25 deg to 45 deg for a CD patient during
wax try-in what would need to be adjusted for the denture (answer choices were something like
Compensating curve, incisal guidance, reducing cusp heights, increase cusp heights I think the
correct answer is Increase cusp heights)
A circumferential clasp is composed of
o A rought wire retentive arm, and a reciprocal bracing arm
You have a patient that has no decay, no discoloration. He has an endontically treated tooth. How would you
restore that tooth
o Place composite in the access opening
o Use a porcelain veneer
o Use a porcelain crown
Know that the most conservative restoration for an endo treated tooth is an onlay
Youre accessing a mandibular molar, but you perforate it through the furcation. How will you treat this?
o Hemisect the tooth
o Bisect the tooth
o Extract the tooth, and place an implant
When would a base metal alloy be preferred over a noble metal alloy
o Fabrication of single crown
o Fabrication of multiunit bridge
o youre going to use a higher gold content for small inlays.
fricative sound is produced by
o the teeth contacting lip
know what a direct retainer (clasp) and indirect retainer (rest) are
what causes a pfm crown to turn green at the margins- copper
at what temperature does necrosis of the bone happen during implant placement know that it happens @ 47
degrees Celsius
you have a patient that needs endo, a post and core, and a crown- why would you do crown lengthening
o to lengthen the crown was an option
o for ferrule effect
what should the distance be between implants (both are implants)- 3 mm
how far away should an implant be placed from the inferior alveolar canal 2 mm
what is the best way to radiographically view structures before an implant- ct
which impression material is the most inaccurate irreversible hydrocolloid
know the disadvantages of polyether
o polyether is dimensionally unstable in the presence of moisture. Polyether is the most rigid and
difficult to remove from the mouth. Demonstrates imbibition.

58

OS
-

the two main causes of clicking in a denture are- excessive vdo and dislodgement
know what dislodges the denture
o one question asked specifically what causes dislodgement- answer was overextension of flanges I think
o the other was how do you check to see if it wont dislodge

protrusive movement

excursive movement

have the patient move their tongue

have the patient practice their speech


Know reduction for functional cusps vs. nonfunctional cusps
o Never grind maxillary lingual cusps (primary holding cusps)
o Secondary holding cusps are mandibular buccal cusps, its ok to grind these if there is balancing side
interferences (inner inclines)
o Cusp tips should be grinded in premature centric, lateral, protrusive contacts
know value, chroma, hue
o value is the single most important determinant . hue should be selected first, then chroma, lastly
value
o you adjust hue by adding pink purple (which will move yellow towards yellow red). You adjust value by
adding a complimentary color (violet on yellow restorations.) staining a porcelain restoration will
reduce the value. It is almost impossible to increase the value (I had several questions on all of this
info).
Hardest to fix if you have a pfm increasing value
Worst way to correct a 1.5 mm space between centrals
o Full crown
o Porcelain veneer
o Acrylic veneer
o Direct composite restoration
Tooth # 30 deviated to the left when placing crown, what is the interference = inner incline of the lingual cusp
Patient wants denture remade, but they present with a white plaque what do you do? = releave the area,
relin and check again, if its still there then biopsy.
You have a patient you did a crown on, the margins were great, its lovely, but she has inflammation at the
gingiva why? = impindged on the biological width?
When do you practice speech when fabricating a denture
o When measuring vdo
o During wax try in
Know INR, PTT, Extrinsic factor
Clopdogril (Plavix) affects the (I got like 5 separate questions concerning how Plavix specifically affects
clotting); INR kept coming up with it, too
o Intrinsic pathway
o Platelet function
o Extrinsic factor
Gave a scenario about an elderly man with a horizontal (I think) impaction, asked why you would consider
extracting his third molar
o The answer was to prevent perio disease in the adjacent tooth
o Indications for removal of impacted teeth:

Prevention of perio disease in teeth adjacent to impacted teeth

Prevention of dental caries

Prevention of pericoronitis

Prevention of root resorption of adjacent teeth prevention of odontogenic cysts and tumors

Treatment of pain of unexplained origin

Prevention of jaw fractures

Facilitation of ortho treatment


o There was another question about a patient who presents with an operculum around an impacted
tooth.. asked if you should extract

Yes to prevent pericoronitis


Know the classifications of impacted teeth
o Mesioangular- least difficult impaction
o Horizontal
o Vertical
o Distoangular- most difficult
Most common post surgery complication:
o Retained fractured root
o Infection
o Bleeding

59

Dry socket (osteitis) is the most common post surgical complication, particularly in the mandible, but
that wasnt an option. Bleeding is a relatively uncommon complication of dental extraction. Causes of
excessive bleeding are injury to the inferior alveolar artery during extraction of a mandibular tooth
(particularly the third molar); a muscular arteriolar bleed from the elevation of a Mucoperiosteal flap
for third molar removal, or bleeding related to the patients hemostasis. Infections are uncommon in
healthy patients. Whenever a Mucoperiosteal flap is elevated for surgical extraction, there is the
possibility for a subperiosteal abscess. So all surgical flaps should be irrigated liberally prior to
suturing.
I didnt have any questions on lefort fracture
I did have that repeat that everyone has whats the benefit of a vertical ramus osteotomy vs BSSO
o Vertical might be preferred, because it is less likely to damage the inferior alveolar nerve
o Vertical body osteotomies involve extracting mandibular teeth bilaterally (usually bicuspids). A piece of
bone is also removed from the mandible and you slide everything back. Used for prognathism
o BSSO: the mandible is split sagitally and can either be used to advance the mandible and cure
Retrognathia, or to set the mandible back, and treat Prognathia. This is the standard procedure used
today.
Know distraction osteogenesis and osteogenesis
o Distraction osteogenesis: involves the cutting an osteomy to separate segments of bone and the
application of an appliance that will facilitate the gradual and incremental separation of bone
segments. Used for patients with cleft lip and palate as well as other deformities of the facial skeleton
I had quite a few questions on biopsy
o Know that you do an incisional biopsy when a lesion is large (>1 cm), is polymorphic, suspicious for
malignancy, or in an anatomic area with high morbidity
o Excisional biopsy: for smaller lesions, (<1 cm) that appear benign and on small vascular and
pigmented lesions. It entails the removal of the entire lesion and a perimeter of surrounding
uninvolved tissue margin.
o Oral brush cytology: used to detect cancerous and precancerous lesions. May be useful for monitoring
or screening lesions in an adjunctive role for observation
What causes stridor- laryngospasm
If you have a pregnant patient, place her in the on her side, if you place her on her back it will put pressure on
the inferior vena cava.
Want to place a patient who has fainted in the trendelenburg position- feet elevated, patient in supine position
(these had several questions on how to place a pregnant patient, one on trendelenburg position)
know when to premedicate
o congenital septal defect
o mitral valve prolapse
most commonly impacted tooth (3rd molars werent an option)- max canine
you have a hematoma 4 days after extraction, whats the cause
o

Perio
-

What is the best brushing technique


o Sulcular (same as bass)
o Charters
o Widman
o stillman
An allograft is:
o Decalcified freeze dried cadaver bone
o Decalcified freeze dried bovine bone
o hydroxyapatite
At what angulation should you hold a scaler
o 45-90 degrees
o 90-120
know how to treat ANUG (had 4 questions on ANUG).
o One question involved the characteristics of nug. Asked, which one isnt a feature of NUG

Fetid odor

Dehiscences

Affected interdental papilla

Pain

The essential components of nug are interdental gingival necrosis, often described as
punched out papillae, pain, bleeding. Variable features include a fetid odor,
lymphadenopathy, fever, and malaise.
o Female patient with no systemic diseases

The treatment for nug, nup includes debridement, hydrogen peroxide (or clorhexidine ) rinses,
and antibiotic therapy (pen V) if there is systemic involvement (fever, malaise, and
lymphadenopathy). Patients with HIV associated NUG require gentle debridement and
antimicrobial rinses.

60

Know when and when not to give antibiotics I had a lot of questions on treating nug
IgG, what does it act on
o Mast cells
o Antigen
Exception question about a smoker question. Asked which was not true for smokers.- increased BOP (smokers
have decreased GCF flow, and bleeding on probing, increased oxidative burst, increased neutrophil chemotaxis
phagocytosis)
When doing a gingivectomy, where do you place the internal incision
o Apical to the crest
o At the cej
o At the MJG
o A beveled incision is made apical to the pocket depth the tissue is removed, the area debrided and
surgical pack placed. Healing is by secondary intention with the formation of a protective clot,
epithelial migration and CT repair.
o Surgical gingivectomy is performed to eliminate suprabony pockets, gingival enlargements, or
suprabony periodontal abscesses. A gingivectomy shouldnt be performed if osseous recontouring is
needed, if the bottom of the pocket is apical to the MGJ, if there is inadequate attached gingiva, or if
there is an esthetic concern..
Know perio treatment planning phases
o Preliminary or emergency extraction of hopeless teeth
o Nonsurgical (phase I therapy)- controlling bacteria, sc/rp
o Surgical (phase II therapy)- placement of implants, grafts, endo
o Restorative (phase III)- placement of final restorations and fixed and removable appliances
o Maintenance (phase IV) evaluation of oral hygiene, status, presence or absence of local factors, and
conditions of the periodontium (pocket depths, attachment levels, mobility, occlusion.)
Know the probing pocket depth is the distance from the gingival margin to the base of the pocket detected
with the probe. Clinical attachment loss (the distance from the CEJ to the base of the pocket detected with a
probe.
You have a man molar with a large amalgam restoration on it. Theres a class III furcation that is 5 mm from
the apex. How would you treat
o Hemisect
o Bicuspidization
o Extract and implant
Need to get access to bone, what flap will you do- Mucoperiosteal flap

Radiology/OP
target metal in xrays:
o tungsten
o lead
the face is divided into
o fifths (Not thirds???)
know the lesions of the tongue had one on the lateral border of the tongue (this was a photo, showed a white
plaque along the posterior lateral border of the tongue.)
o know that the lateral posterior border of the tongue is most common intraoral site for SCC. The lower
lip is most common site for oral cancer.
(picture question) had one question where they should an ulcerative lesion (about 2 cm, red, on midline of
tongue) and they told me there was a similar one on the palate.
o Syphilis
o Gonorrhea
o Kaposi sarcoma
o candidiasis
o Syphilis is caused by contact with patient infected with treponema palidum. The primary lesion is a
chancre; the secondary lesions are oral mucous patches, condyloma latum, maculopapular rash, and
tertiary lesions (gummas, CNS/cardiovascular involvement). Tertiary syphilis most likely occurs on the
palate and the tongue Gonorrhea has oral pharyngitis as a manifestation, but is rarely seen.
(picture question) Showed a tooth (#18) with a fractured gold restoration, asked what the lesion adjacent of
the tooth was.
o Papilloma
o Fibroma
o Endo abscess
o I thought the nodule was from the endodontically involved lesion. It was a smooth, pink sessile mass,
looked like an abscess to me.
o But, traumatic fibroma is a reactive lesion caused by trauma to oral mucous membranes. Its painless,
broad based swelling that is lighter than surrounding tissue, frequently found on the buccal mucosa,

61

lateral border of the tongue, and lower lip. Id still go with that being an endo tooth because of the
symptoms the patient reported, but know indications for both
Asked about a patient who had blockage in stensens duct, described it as looking like a sausage
radiographically what causes thiso Chronic sialodochitis
Patient had a blue lesion on the floor of the mouth ranula
o Ranula is term for mucoceles that occur in the floor of the mouth. It appears as a blue, dome shaped,
fluctuant swelling in the fom. Usually located lateral to the midline. Treat by removing the sublingual
gland, or marsupialization
Patient bit her lip, then had a lesion on her lower lip that was blue- mucocele.
o Mucocele is a common lesion of the oral mucosa that results from rupture of a salivary gland and
spillage of mucin into the surrounding soft tissues. This spillage is often the result of local trauma,
although there is no known history of trauma in many cases. Not a true cyst because it lacks epithelial
lining
Papillon lefevreHad a question on aphthous lesions
o Minor aphthous ulcers one to several painful oval ulcers <.5 cm. most common type, duration of 710 days
o Major aphthous ulcers- up to deep crater form ulcers > .5 cm, very painful and possibly debilitating
may take several weeks to heal
o Herpetiform aphthous ulcers- recurrent crops of minor aphthae, painful, 1-2 weeks to heal. May be
found on any mucosal surface, same cause as other aphthae (not viral)
Had a question about secondary herpes and the palate
o Know that primary herpes affects the perioral area, especially the gingiva
o Secondary herpes affects the hard palate, lips, and gingiva palate is only secondary
What causes widening of the pdl
o Osteosarcoma
o Other things that cause widening of the pdl but werent options, scleroderma,
Described a nasolabial duct cyst no picture
Between pemphigus and pemphigoid which one affects the basement membrane- mucous membrane
pemphigoid causes autoantibodies directed against the basement membrane
Had a question about cancer to the palate. They described a roughened growth to the palate, asked you to
identify it.
o Most common salivary malignancy in minor and major glands is the Mucoepidermoid carcinoma, palate
is the most common intraoral site
o Polymorphous low grade adenocarcinoma is the second most common minor salivary gland
malignancy, palate is most common site
o Adenoid cystic carcinoma is a high-grade salivary malignancy, palate is most common site.
Know dentinogenesis imperfecta, had a question that asked which disorder shouldnt be on the differential for
dentinogenesis imperfecta
o Amelogenesis
o Ectodermal dysplasia
o Dentinal dysplasia
What would make you suspect bulimia erosion on the lingual surface of incisors
If a disorder has nevoid basal cell carcinoma, what else would you expect to see OKC
Had some cleft palate questions
Lip/Cleft palate incidence
o 1:700
what are the causes of cleft palate
o environmental
o genetic
o multifactorial
young girls, radiolucent lesion with white specks- AOT
had a question on acromegaly. Asked what feature is most common in terms of face shapeo excessive maxilla
o excessive mandible
picture of inverted y on a pa

62

o the anterior wall of the max sinus makes the inverted y landmark (3, 4, in the picture)
recognize the zygomatic process on a panoramic radiograph

1.
2.
3.
4.
5.
6.
7.
8.
9.

articular tubercle, temporal bone


Zygoma
Zygomatic process of the maxilla
Pterygomaxillary fissure
Floor of orbit
Anterior aspect of inferior concha
Nasal septum
Anterior nasal spine
Floor of max sinus

63

10. Maxillary left third molar (developing)


11. Ear lobe
12. Cervical vertebral body
3 questions on condensing osteitiso aka focal sclerosing osteomyelitis. Involves bony sclerosis around the roots of teeth with associated
pulpal infection (ex. Pulpitis or pulpal necrosis,) commonly seen in kids and young adults.
o Clinically appears as a localized radiopaity, doesnt mimic a periapical granuloma radiographically.
Doesnt connect with root
Had a question on periapical COD- know its black women, vital teeth

pharm
drug A works on a small dose of drug B to increase drug Bs efficacy. The same drug A works on a large dose of
B with no effect. What Is this called
o partial agonist
o antagonist
o other options
o agonists- have a purely intrinsic activity
o antagonists- have no intrinsic activity

intrinsic activity is the maximal effect of a drug


side effect of anticancer medication- bone marrow suppression
if peripheral arterioles are acted upon, what adrenergic receptor on those arterioles is being affected
o alpha
o beta
o alpha 1 receptor agonists cause vasoconstriction, urinary retention, Mydriasis
o alpha 2 receptors cause hypotension by reducing sympathetic outflow from the CNS
o beta 1 receptor agonists increase cardiac rate and force of contraction, this positive inotropic and
chronotropic actions (increased heart rate)
o beta 2 receptor agonists cause dilation of skeletal muscle, blood vessel and bronchi relaxation
(bronchodilation, vasodilation)
patient takes albuterol everyday, what do they have? asthma
keep in mind Pen V is the antibiotic of choice for odontogenic infections
o narrow spectrum antibiotics are preferable over broad spectrum because they have a lesser chance of
altering the normal flora with its associated symptoms
an asthmatic patient should avoid which drug? NSAID. Give them acetaminophen
why doesnt anesthesia work when you have an infection- decreased pH leads to a more ionized form
what drug combination will include a stage 2 drug- gave acetaminophen with various drug options.
I had a lot of epi questions
o Propanolol reduces o2 demand by preventing chronotropic responses to endogenous epi, emotions and
exercise (how it treats angina)
o Graves disease leads to excess circulating thyroid hormone which results in a hypermetabolic state
with heightened sympathetic activity which combined with injected epi could result in a hypertensive
crisis
o The use of epi for local hemostasis during surgery might result in cardiac arrhythmia
o Epi reverses anaphylaxis, causes vasoconstriction, and bronchodilation
Zantac is an H2 blocker used for gastric upsetKnow promethazine is used as a preoperative medication for sedation with antiemetic properties, lil wayne
drinks this as his sleepy juice!
Know the minimum dose of steroid you need to get addisons
o 20 mg for 2 weeks every 2 years
o 200 mg for 2 weeks every 2 years (I think its this one???)
o (these were the 2 largest values)
Sulfonylureas moa
o MOA: close K channels in cell membranes, stimulate release of insulin from pancreas, increase
sensitivity of target organs to insulin.
Know symptoms of CHF
Know moa of digitalis
o Digitalis acts by decreasing the AV conduction
Super straightforward question about a teratogen
All of these treat angina except
o Calcium channel blockers
o Beta blockers
o Diuretics
o Nitrates/ nitrites
Know moa of lidocaine..

64

o
o

The non-ionized or free base form can penetrate tissue membranes


Inflamed tissue has a lower than normal pH which decreases the amount of non-ionized forma
available to penetrate

Peds/ortho
the most effective endo test for a primary tooth- endo ice
which tooth is most prone to caries between first max molar, second max molar, 1 st man molar, 2nd man
molar
o still dont know.. Im gonna guess one of the first molars since theyre always indicated for sealants..
also, the canines are prone to caries idk
if the tooth has a deformation of size (peg lateral, Microdontia) what phase of development was affected
o Initiation phase- anodontia and supernumerary teeth
o Cap stage period of proliferation, differentiation, and morphogenesis dens in dente
o Bell stage problems of Histodifferentiation and morphodifferentiation- amelogenesis imperfecta,
Macrodontia, Microdontia
o Apposition- cells begin to deposit dental tissues (enamel, dentin, cementum)- enamel dysplasia,
enamel hypoplasia, concrescence
I had a lot of behavioral peds questions
had a lot of fluoride questions
o range of fluoride concentration in water - 0.7-1.2
o what is the youngest age you would start giving fluoride supplementation ( I based my answer on the
peds fluoride chart)
o whats the standard fluoridation of water

.5

1.5

2
you have a patient that is in crossbite thats not skeletal. What caused it?
o Inadequate space
o Too narrow a maxillary arch
what facial profile would you expect in a patient with trisomy 21
o Pronounced mandible
o Normal mandible
o Pronounced maxilla
o Midface hypoplasia
Know when teeth calcify
Relying on a big brother to set an example for a child iso Modeling
Tooth looks submerged what would you suspect
o Ankylosis
Which type of fluoride isnt used in toothpaste- apf
What kind of fluoride rinse would you give a school age child as a supplement
o Weekly rinse NaF .2% this is right
o Daily rinse NaF .05 percent this is right as well but Mosbys said that is easier to do weekely rinses
than daily
o Daily rinse NaF.2% this is too much F so is wrong
Biggest problem if tooth doesnt erupt, or erupts prematurely (which tooth is the biggest problem in terms of
the arch) I said canine
Which of these is the most desirable storage medium
o Blood
o Water (least desirable)
o Soy milk
o Hanks balanced salt solution
The sagito-occipital complex is fused by
o Intramembranous bone,
o Fibrous suture
o The cranial base (ethmoid, sphenoid, and occipital bones) at the base of the skull are formed initially in
cartilage, and later transformed into bone by endochondral ossification. As ossification occurs, three
bands of cartilage remain, which are important growth centers called synchodroses/ each
synchodrosis acts like a two sided epiphyseal plate with a growth cartilage in the middle and bands of
maturing cartilage cells extending in both direction that will be replaced by bones.
Know at what age the mandibular symphesis fuses
o Google says 6-9 months
If you superextrude posterior teeth, how will it affect the anterior teeth ( it will open the anterior bite)
o Relative deep bite
o Clinical deep bite

65

o Some other options


Mouth breathers know how their mouths are messed up
o Lower occlusal plane
o Narrow maxillary arch
Restorative concern for primary molars- I think it has to do more with thin dentin, enamel, and pulp
morphology.
o divergent roots,
o wide occlusal table,
o shallow pits
patient management
know chi test, z test, t test.
Had a question about upcoding in terms of billing
Had a question about Ins company changing a Class II code to a Class I code downcoding, yes?
Know the difference between sympathy and empathy.
o For one question they gave 2 sentences, asked which one was sympathy, which one was empathy
o For another question (exception question) asked which one wasnt a characteristic of empathy
Question about upcoding
Know specificity vs. sensitivity
o True Positive (TP) = Those who have the disease and are correctly identified as having the
disease
o False Negative (FN) = those who are incorrectly classified as NOT having the disease MISSED
DIAGNOSIS
o True Negative (TN) = those who do not have the disease and are correctly identified as not
having the disease
o False Positive (FP) = those who have the disease who are NOT identified by the result
-

o
o
o

Sensitivity vs. Specificity


Sensitivity is defined as the percent of persons WITH the disease who are CORRECTLY
CLASSIFIED as having the disease (those who have the disease)
Sensitivity = TP/(TP + FN) x 100%

Specificity = defined as the percent of people WITHOUT the disease who are CORRECTLY
CLASSIFIED as NOT having the disease (those who do not have the disease)
o Specificity = TN/(TN + FP) x 100%
If you clean an inanimate object and you dont destroy the spores what is it called = disinfection
Know null hypothesis = there is no difference between two values p is less than 0.05
Know cohort vs. experimental
types of studies: epidemiological studies can be organized into three categories
o descriptive: used to quantify disease status in the community. The major parameters of interest are
prevalence and incidence

prevalence indicates what proportion of a population is affected by a condition at a given


point in time. It is expressed as percentages. Ex. The prevalence of periodontal disease
among 100,000 adolescents was 5%

prevalence = Number of people with the disease


total number of people at risk

incidence: indicates the number of new cases that will occur within a population over a
period of time (ex. The incidence of people dying of oral cancer is 10% per year in men aged
55-59 in our community
o incidence= number of new cases of the disease/ total number of people at risk
o analytical epidemiology: used to determine the etiology of a disease. The researcher tries to
establish a causal relationship between factors and disease. Three study designs are used: cross
sectional stud, case control study, and cohort study (prospective and retrospective)

cross sectional study: a study in which the health conditions in a group of people in a cross
section (population) is assess at one time.

Its quick and inexpensive, but the potential to contribute to a judgment of causation is
limited because it cant determine whether the outcome occurred before the behavior
in question, or if it developed because of another cause

Case control study: people with a condition (cases) are compared with people without it
(controls) but who are similar in other characteristics.

Cohort study

Prospective cohort study: a general population is followed through time to see who
develops the disease, and then the various exposure factors that affected the group
are evaluated. Following the group over a period of time, the investigators describe
the prevalence of outcomes
o

66

Retrospective cohort study: used to evaluate the effect that a specific exposure has
had on a population. The investigator chooses or defines a sample of the subjects who
had the outcome of interest and looks back at risk factors that may have predicted the
subsequent outcome
Experimental: compare the incidence of disease and the side effects between the groups in the study
to draw inferences about safety and efficacy of the treatment under investigation

Well designed trials use a double blind design (neither subject nor investigator knows which
group a subject belongs)

day 2
Dont remember my specific cases as much. I had 2 ortho cases, 3 mostly prosth cases
Ortho case 1o Involved a young man, asked for his face shape, asked about his teeth (premolars were submerged
and 3rd molars were impacted). He was taking asthma medications. Asked what you wouldnt give
him, answers were NSAIDs, asked how you would fix his crossbite
Ortho case 2o Young girl, also asthmatic, asked her face shape, asked how you would fix her canines (ectopically
erupted),
Prosth caseo All of them were pretty much the same it seemed.. one of them had a cantilever and there were
retained roots underneath, there were some questions on the cantilever and the roots. One of my
guys had PTSD, asked some questions about that. Asked a question about CHF they were all on
hypertensive drugs, they all had multiple edentulous spans, asked how you would treat/fix existing
dentition. Know when to premedicate I had a question on that for each of them.
For my cases, my advice to be to know about crossbites, and when to use a palatal expander, know when to
premedicate. My questions involved either ortho, prosth, os, or implants. No real op questions.

Oct 2013

2) Name all Symptoms of thyrotoxicosis and what it is? Sweating, heat intolerance, tachycardia, warm,
thin, soft, moist skin, tremor and exophthalmos.
3) What is neurapraxia EXACTLY?
4) Pregnant woman in supine position? Abdominal aorta, IVC (inferior vena cava), placenta. And
something about Hip raising?
5) Caf-aulat spots and freckles question. MENS? Neurofibrosis? Or Peutz-Jeghers Syndrome
6) 10 month ago, hip replacement surgery was done with a pt who presents to your dental clinic. I dont
remember what kind of dental procedure was asked, but the choices asked about. A) medical
consultation from primary physician to ask what kind of antibiotic B) No antiobiotic needed C)
Amoxcillin
7) Thumb/Finger sucking appliance does what kind of conditioning to the child? Aversive? Cognitive?
8) Had a question about an appliance that would fix thumb sucking habit and posterior crossbite? Can a
tongue crib do both? Other options were quad helix, twin block, and either Nance or TPA.
9) Preparing a veneer, in middle 3rd, how much do you reduce .5 mm You must know this Pg.454
10) Know your indications for a onlay AND inlay
11) Too much Acetaminophen liver dz
12) Hypoglycemia know what happens in these patients= hypoglycemia (see Chapter 17), including
tachycardia, perspiration, weakness, nausea, vomiting, headache, convulsions, and coma.
13) Most effective way you tx Grand Mal? Phenytoin?
14) Treatment for child who has petit mal seizure in your chair IV diazepam (assuming you can drop an IV
I didnt pick this answer for that reason. I chose something like keep the patient from hurting
himself)
15) Patient needs a pain killer that lasts for 8 hours, which will you give? Naproxen
16)

67

May 2015 I had a TON of questions from the list above. Below I added a few that I did not remember being on the list
or that tripped me up a little from the actual wording of the question to what was posted on here. Good luck!
What phase is highest creep phase in amalgam? Tin-silver, Copper-tin, Tin-mercury, mercury-silver
Pregnant patient syncope? Right hip up, left hip up, Trendelenburg position
Primary reason for anterior max incisor endo failure? Cleaning and shaping, inadequate obturation, improper
sterilization
Which opiod is a agonist and antagonist? Hydrocodone, Meperidine, Oxycodone, Butorphanol.
Match opiod and correct thing? Hydrocodne-Antagonist & Agonist, Meperidine- Synthetic agonist,
Antibiotic for B-lactamase producing staph? Erythomyosin, Imipenem
Which is a vasodialator? Histamine, Histadine, Diphenalhydramine
What is best for an anxious pregnant patient? Nitrous, Diazepam, Triazelam
What is associated with angular chelitis? Candida, Vit deficiency
Maxillary tooth with fluted mesial root making it hard to scale? 1st PM, 2nd PM, 1st molar, 2nd Molar
Fluoride supplement for child 9 yr old? 1mg PER DAY!!!!!!!!!! FML
What is hardest to change of PFM? Hue, Chroma, Increase value, decrease value
Highest child caries rate? Blacks, Native Americans, Whites
Highest oral cancer? Black males, White males, Asian females, Mexican females
What doesnt cause macroglossia? Amyloid deposits from MM, Hypothyroid, ..
Signs of Opiod overdose? Constipation, mydriasis, insomnia, irritabiliy
Signs of Opiod abuse? Diarrhea, chronic cough
What does Quinidine do? Ventricle arythmia
Implant to implant distance? 3mm
Know what a damn crown and prep looks like and indications for and what wall prevents lingual tipping? I put
proximal lingual wall.
Impression material with best dimensional stability? Vynil Polysiloxane, Polyether
Dentist wants to switch from some xray to farther away (from 8inches to 16inches). How much longer is exposure
time? 2 times, 4, 6, 8

Similar question, but asked how it affected the intensity of the beam doubling the distance results in a beam
that is as intense
Distance of object to x ray source in CT? 5 ft, 6ft, 50cm, 60cm
Posterior teeth hit when protrusive movement with denture. How does dentist fix? Adjust compensating curve,
increase incisal guide,..
What tooth refers pain to ear? Posterior Maxillary, posterior mandible, Md premolars, Mx premolars
What does water contamination do to Amalgam? Weaken, expansion, prevents polishibility
Primary reason for amalgam failure? Under triteration, not condensed enough, undermined enamel
Patient has Petite mal seizure, what do you do? Do nothing, keep patient from harm, diazepam.
Mandibular first molar calcification complete? 2.5-3yrs, 4-5yrs
Natural supplement that increases anticoagulant effects? Black licorice, st. johns wart, chamomile

68

First sign of Cavernous sinus infection? Vision problems, Opth, ..


Antifungal that comes in a form called troches? Clotrimazole, Ketakanozal, Itraconazole, flucanazole

69

Вам также может понравиться